为了正常的体验网站,请在浏览器设置里面开启Javascript功能!

Feifei_Logic

2018-09-10 50页 doc 230KB 33阅读

用户头像

is_361945

暂无简介

举报
Feifei_Logic 《费费逻辑宝典》题目版 · 费费逻辑宝典全集(共135题)是目前公认的非常好的GMAT逻辑复习补充材料,这是费费从最新LSAT逻辑中摘出的他认为最具有代表性也是最符合GMAT逻辑原则的135题。 · 本套练习题共135题,根据牛人建议:为锻炼Pace,每18题划分为一组,限时30分钟完成。 Test 1 1. Although 90 percent of the population believes itself to be well informed about health care, only 20 percen...
Feifei_Logic
《费费逻辑宝典》题目版 · 费费逻辑宝典全集(共135题)是目前公认的非常好的GMAT逻辑复习补充材料,这是费费从最新LSAT逻辑中摘出的他认为最具有代表性也是最符合GMAT逻辑原则的135题。 · 本套练习题共135题,根据牛人建议:为锻炼Pace,每18题划分为一组,限时30分钟完成。 Test 1 1. Although 90 percent of the population believes itself to be well informed about health care, only 20 percent knows enough about DNA. So apparently at least 80 percent of the population does not know enough about medical concepts to make well-informed personal medical choices or to make good public policy decisions about health care. The argument’s reasoning is questionable because the argument fails to demonstrate that (A) those people who can understand news stories about DNA are able to make well-informed personal medical choices (B) more than 20 percent of the population needs to be well informed about health care for good public policy decisions about health care to be made (C) one’s being able to make well-informed personal medical choices ensures that one makes good public policy decisions about health care (D) an understanding of DNA is essential to making well-informed personal medical choices or to making good public policy decisions about health care (E) since 90 percent of the population believes itself to be well informed about health care, at least 70 percent of the population is mistaken in that belief. 2. During the 1980’s, Japanese collectors were very active in the market for European art, especially as purchasers of nineteenth-century Impressionist paintings. This striking pattern surely reflects a specific preference on the part of many Japanese collectors for certain aesthetic attributes they found in nineteenth-century Impressionist paintings. Which one of the following, if true, most strongly supports the explanation above? (A) Impressionist paintings first became popular among art collectors in Europe at the beginning of the twentieth century. (B) During the 1980’s, the Japanese economy underwent a sustained expansion that was unprecedented in the country’s recent history. (C) Several nineteenth-century Impressionist painters adopted certain techniques and visual effects found in Japanese prints that are highly esteemed in Japan. (D) During the 1960s and the 1970s, the prices of nineteenth-century Impressionist paintings often exceeded the prices of paintings by older European masters. (E) During the 1980’s, collectors from Japan and around the world purchased many paintings and prints by well-known twentieth-century Japanese artists. 3. Because of the recent recession in Country A most magazines published there have experienced decreases in advertising revenue, so much so that the survival of the most widely read magazines is in grave doubt. At the same time, however, more people in Country A are reading more magazines than ever before, and the number of financially successful magazines in Country A is greater than ever. Which one of the following, if true, most helps to resole the apparent discrepancy in the information above? (A) Most magazines reduce the amount they charge for advertisement’s during a recession. (B) The audience for a successful television show far exceeds the readership of even the most widely read magazine. (C) Advertising is the main source of revenue only for the most widely read magazines; other magazines rely on circulation for their revenue. (D) Because of the recession, people in Country A have cut back on magazine subscriptions and are reading borrowed magazines. (E) More of the new general interest magazines that were launched this year in Country A have survived than survived in previous years. 4. The gray squirrel, introduced into local woodlands ten years ago, threatens the indigenous population of an endangered owl species, because the squirrels’ habitual stripping of tree bark destroys the trees in which the owls nest. Some local officials have advocated setting out poison for the gray squirrels. The officials argue that this measure, while eliminating the squirrels, would pose no threat to the owl population, since the poison would be placed in containers accessible only to squirrels and other rodents. Which one of the following, if true, most calls into question the officials’ argument? (A) One of the species whose members are likely to eat the poison is the red squirrel, a species on which owls do not prey. (B) The owls whose nesting sites are currently being destroyed by the gray squirrels feed primarily on rodents. (C) No indigenous population of any other bind species apart from the endangered owls is threatened by the gray squirrels. (D) The owls that tare threatened build their nests in the tops of trees, but the gray squirrels strip away back from the trunks. (E) The officials’ plan entails adding the poison to food sources that are usually eaten by rodents but not by other animals. Questions 5-6 Sales manager: Last year the total number of meals sold in our company restaurants was much higher than it was the year before. Obviously consumers find our meals desirable. Accountant: If you look at individual restaurants, however, you find that the number of meals sold actually decreased substantially at every one of our restaurants that was in operation both last year and the year before. The desirability of our meals to consumers has clearly decreased, given that this group of restaurants---the only ones for which we have sales figures that permit a comparison between last year and the year before---demonstrates a trend toward fewer sales. 5. If the sales figures cited by the accountant and the sales manager are both accurate, which one of the following must be true? (A) The company opened at least one new restaurant in the last two years. (B) The company’s meals are less competitive than they once were. (C) The quality of the company’s meals has not improved over the last two years. (D) The prices of the company’s meals have changed over the past two years. (E) The market share captured by the company’s restaurants fell last year. 6. Which one of the following, if true, most seriously calls into question the accountant’s argument? (A) The company’s restaurants last year dropped from their menus most of the new dishes that had been introduced the year before. (B) Prior to last year there was an overall downward trend in the company’s sales. (C) Those of the company’s restaurants that did increase their sales last year did not offer large discounts on prices to attract customers. (D) Sales of the company’s most expensive meal contributed little to the overall two-year sales increase. (E) Most of the company’s restaurants that were in operation throughout both last year and the year before are located in areas where residents experienced a severe overall decline in income last year. 7. Problems caused by the leaching of pollutants from dumps and landfills are worst in countries with an annual per capita economic output of $4,000 to $5,000, and less severe for considerably poorer and considerably richer countries. This is so because pollution problems increase during the early stages of a country’s industrial development but then diminish as increasing industrial development generates adequate resources to tackle such problems. Therefore, problems caused by such leaching in Country X, where the annual per capita economic output is now $5,000, should begin to diminish in the next few years. Which one of the following is an assumption on which the argument depends? (A) Within the next few years, Country X will impose a system of fines for illegal waste disposal by its industrial companies. (B) Countries surrounding Country X will reduce the amount of pollution that their factories release into the air and water. (C) Industrial development in Country X will increase in the next few years. (D) Country X will begin the process of industrialization in the next few years. (E) No other country with a similar amount of industrial development has pollution problems that are as severe as those in Country X. 8. The stable functioning of a society depends upon the relatively long-term stability of the goals of its citizens. This is clear from the fact that unless the majority of individuals have a predictable and enduring set of aspirations, it will be impossible for a legislature to craft laws that will augment the satisfaction of the citizenry, and it should be obvious that a society is stable only if its laws tend to increase the happiness of its citizens. The claim that a society is stable only if its laws tend to increase the happiness of its citizens plays which one of the following roles in the argument? (A) It is the conclusion of the argument. (B) It helps to support the conclusion of the argument. (C) It is a claim that must be refuted if the conclusion is to be established. (D) It is a consequence of the argument. (E) It is used to illustrate the general principle that the argument presupposes. 9. Astronauts who experience weightlessness frequently get motion sickness. The astronauts see their own motion relative to passing objects, but while the astronauts are weightless their inner ears indicate that their bodies are not moving. The astronauts’ experience is best explained by the hypothesis that conflicting information received by the brain about the body’s motion causes motion sickness. Which one of the following, if true, provides the strongest additional support for the hypothesis above? (A) During rough voyages ship passengers in cabins providing a view of the water are less likely to get motion sickness than are passengers in cabins providing no view. (B) Many people who are experienced airplane passengers occasionally get motion sickness. (C) Some automobile passengers whose inner ears indicate that they are moving and who have a clear view of the objects they are passing get motion sickness. (D) People who have aisle seats in trains or airplanes are as likely to get motion sickness as are people who have window seats. (E) Some astronauts do not get motion sickness even after being in orbit for several days. 10. In order to pressure the government of Country S to become less repressive, some legislators in Country R want to ban all exports from R to S. Companies in R that manufacture telecommunication equipment such as telephones and fax machines have argued that exports of their products should be exempted from the ban, on the ground that it is impossible for a country to remain repressive when telecommunication equipment is widely available to the population of that country. Which one of the following is an assumption on which the argument given by the manufacturers depends? (A) The government of S has recently increased the amount of telecommunication equipment it allows to be imported into the country. (B) The telecommunication equipment that would be imported into S if the exemption were to be granted would not be available solely to top development officials in S. (C) A majority of the members of R’s legislature do not favor exempting telecommunication equipment from the ban on exports to Country S. (D) Of all exports that could be sent to Country S, telecommunication equipment would be the most effective in helping citizens of S oppose that country’s repressive government. (E) Without pressure from Country R, the government of S would be able to continue repressing its citizens indefinitely. 11. The average cable television company offers its customers 50 channels, but new fiber-optic lines will enable telephone companies to provide 100 to 150 television channels to their customers for the same price as cable companies charge for 50. Therefore, cable companies will be displaced by the new companies within a few years. Which of the following, if true, most helps to strengthen the argument? (A) The initial cost per household of installing new fiber-optic television service will exceed the current cost of installing cable television service. (B) The most popular movies and programs on channels carried by cable companies will also be offered on channels carried by the fiber-optic lines owned by the telephone companies. (C) Cable television companies will respond to competition from the telephone companies by increasing the number of channels they offer. (D) Some telephone companies own cable companies in areas other than those in which they provide telephone services. (E) The new fiber-optic services offered by telephone companies will be subject to more stringent governmental programming regulations than those to which cable companies are now subject. 12. The only physical factor preventing a human journey to Mars has been weight. Carrying enough fuel to propel a conventional spacecraft to Mars and back would make even the lightest craft too heavy to be launched from Earth. A device has recently been invented, however, that allows an otherwise conventional spacecraft to refill the craft’s fuel tanks with fuel manufactured from the Martian atmosphere for the return trip. Therefore, it is possible for people to go to Mars in a spacecraft that carries this device and then return. Which one of the following is an assumption on which the argument depends? (A) The amount of fuel needed for a spacecraft to return from Mars is the same as the amount of fuel needed to travel from Earth to Mars. (B) The fuel manufactured from the Martian atmosphere would not differ in composition from the fuel used to travel to Mars. (C) The device for manufacturing fuel from the Martian atmosphere would not take up any of the spaceship crew’s living space. (D) A conventional spacecraft equipped with the device would not be appreciably more expensive to construct than current spacecraft typically are. (E) The device for manufacturing fuel for the return to Earth weighs less than the tanks of fuel that a conventional spacecraft would otherwise need to carry from Earth for the return trip. 13. In 1712 the government of Country Y appointed a censor to prohibit the publication of any book critical of Country Y’s government; all new books legally published in the country after 1712 were approved by a censor. Under the first censor, one half of the book manuscripts submitted to the censor were not approved for publication. Under the next censor, only one quarter of the book manuscripts submitted were not approved, but the number of book manuscripts that were approved was the same under both censors. If the statements in the passage are true, which one of the following can be properly concluded from them? (A) More books critical of Country Y’s governments were published before the appointment of the first censor than after it. (B) The first censor and the second censor prohibited the publication of the same number of book manuscripts. (C) More book manuscripts were submitted for approval to the first censor than to the second. (D) The second censor allowed some book manuscripts to the published that the first censor would have considered critical of Country Y’s government. (E) The number of writers who wrote unpublished manuscripts was greater under the first censor than under the second. 14. If the government increases its funding for civilian scientific research, private patrons and industries will believe that such research has become primarily the government’s responsibility. When they believe that research is no longer primarily their responsibility, private patrons and industries will decrease their contributions toward research. Therefore, in order to keep from depressing the overall level of funding for civilian scientific research, the government should not increase its own funding. Which one of the following is an assumption on which the argument relies? (A) Governments should bear the majority of the financial burden of funding for civilian scientific research. (B) Any increase in government funding would displace more private funding for civilian scientific research than it would provide. (C) Private donations toward research are no longer welcomed by researchers whose work receives government funding. (D) Civilian scientific research cannot be conducted efficiently with more than one source of funding. (E) funding for civilian scientific research is currently at the highest possible level. 15. Dental researcher: Filling a cavity in a tooth is not a harmless procedure: it inevitably damages some of the healthy parts of the tooth. Cavities are harmful only if the decay reaches the nerves inside the tooth, and many cavities, if left untreated, never progress to that point. Therefore, dentists should not fill a cavity unless the nerves inside the tooth are in imminent danger from that cavity. Which one of the following principles, if valid, most strongly supports the researcher’s reasoning? (A) Dentists should perform any procedure that is likely to be beneficial in the long term, but only if the procedure does not cause immediate damage. (B) Dentists should help their patients to prevent cavities rather than waiting until cavities are present to begin treatment. (C) A condition that is only potentially harmful should not be treated using a method that is definitely harmful. (D) A condition that is typically progressive should not be treated using methods that provide only temporary relief. (E) A condition that is potentially harmful should not be left untreated unless it can be kept under constant surveillance. 16. The axis of Earth’s daily rotation is tilted with respect to the plane of its orbit at an angle of roughly 23 degrees. That angle can be kept fairly stable only by the gravitational influence of Earth’s large, nearby Moon. Without such a stable and moderate axis tilt, a planet’s climate is too extreme and unstable to support life. Mars, for example, has only very small moons, tilts at wildly fluctuating angles, and cannot support life. If the statements above are true, which one of the following must also be true on the basis of them? (A) If Mars had a sufficiently large nearby moon, Mars would be able to support life. (B) If Earth’s Moon were to leave Earth’s orbit, Earth’s climate would be unable to support life. (C) Any planet with a stable, moderate axis tilt can support life. (D) Gravitational influences other than moons have little or no effect on the magnitude of the tilt angle of either Earth’s or Mars’s axis. (E) No planet that has more than one moon can support life. 17. Psychologist: Some astrologers claim that our horoscopes completely determine our personalities, but this claim is false, I concede that identical twins---who are, of course, born at practically the same time---often do have similar personalities. However, birth records were examined to find two individuals who were born 40 years ago on the same day and at exactly the same time---one in a hospital in Toronto and one in a hospital in New York. Personalities of these two individuals are in fact different. Which one of the following is an assumption on which the psychologist’s argument depends? (A) Astrologers have not subjected their claims to rigorous experimentation. (B) The personality differences between the two individuals cannot be explained by the cultural difference between Toronto and New York. (C) The geographical difference between Toronto and New York did not result in the two individuals having different horoscopes. (D) Complete birth records for the past 40 years were kept at both hospitals. (E) Identical twins have identical genetic structures and usually have similar home environments. 18. Modern navigation systems, which are found in most of today’s commercial aircraft, are made with low-power circuitry, which is more susceptible to interference than the vacuum-tube circuitry found in older planes. During landing, navigation systems receive radio signals from the airport to guide the plane to the runway. Recently, one plane with low-power circuitry veered off course during landing, its dials dimming, when a passenger turned on a laptop computer. Clearly, modern aircraft navigation systems are being put at risk by the electronic devices that passengers carry on board, such as cassette players and laptop computers. Which one of the following, if true, LEAST strengthens the argument above? (A) After the laptop computer was turned off, the plane regained course and its navigation instruments and dials returned to normal. (B) When in use all electronic devices emit electromagnetic radiation, which is known to interfere with circuitry. (C) No problems with navigational equipment or instrument dials have been reported on flights with no passenger-owned electronic devices on board. (D) Significant electromagnetic radiation from portable electronic devices can travel up to eight meters, and some passenger seats on modern aircraft are located within four meters of the navigation systems. (E) Planes were first equipped with low-power circuitry at about the same time portable electronic devices became popular. Test 2 19. Jane: Television programs and movies that depict violence among teenagers are extremely popular. Given how influential these media are, we have good reason to believe that these depictions cause young people to engage in violent behavior. Hence, depictions of violence among teenagers should be prohibited from movies and television programs, if only in those programs and movies promoted to young audiences. Maurice: But you are recommending nothing short of censorship! Besides which, your claim that television and movie depictions of violence cause violence is mistaken: violence among young people predates movies and television by centuries. Which one of the following, if true, most strengthens Jane’s argument? (A) The most violent characters depicted in movies and on television programs are adult characters who are portrayed by adult actors. (B) The movies that have been shown to have the most influence on young people’s behavior are those that are promoted to young audiences. (C) The people who make the most profits in the movie and television industry are those who can successfully promote their work to both young and old audiences. (D) Many adolescents who engage in violent behavior had already displayed such behavior before they were exposed violence in movies. (E) Among the producers who make both movies and television programs, many voluntarily restrict the subject matter of films directed toward young audiences. 20. A rise in the percentage of all 18-year-olds who were recruited by the armed services of a small republic between 1980 and 1986 correlates with a rise in the percentage of young people who dropped out of high school in that republic. Since 18-year-olds in the republic are generally either high school graduates or high school dropouts, the correlation leads to the conclusion that the republic’s recruitment rates for 18-year-olds depend substantially on recruitment rates for high school dropouts. Which one of the following statements, if true, most weakens the argument? (A) A large number of 18-year-old high school graduates were recruited for the republic’s armed services in 1986 than in 1980. (B) Many of the high-technology systems used by the republic’s armed services can be operated only by individuals who have completed a high school education. (C) Between 1980 and 1986 the percentage of high school graduates among 18-year-olds recruited in the republic rose sharply. (D) Personnel of the republic’s armed services are strongly encouraged to finish their high school education. (E) The proportion of recruits who had completed at least two years of college education was greater in 1986 than in 1980. 21. Historian: We can learn about the medical history of individuals through chemical analysis of their hair. It is likely, for example, that Isaac Newton’s psychological problems were due to mercury poisoning; traces of mercury were found in his hair. Analysis is now being done on a lock of Beethoven’s hair. Although no convincing argument has shown that Beethoven ever had a venereal disease, some people hypothesize that venereal disease caused his deafness. Since mercury was commonly ingested in Beethoven’s time to treat venereal disease, if researchers find a trace of mercury in his hair, we can conclude that this hypothesis is correct. Which one of the following is an assumption on which the historian’s argument depends? (A) None of the mercury introduced into the body can be eliminated. (B) Some people in Beethoven’s time did not ingest mercury. (C) Mercury is an effective treatment for venereal disease. (D) Mercury poisoning can cause deafness in people with venereal disease. (E) Beethoven suffered from psychological problems of the same severity as Newton’s. 22. In 1992, a major newspaper circulated throughout North American paid its reporters an average salary paid by its principle competitors to their reporters. An executive of the newspaper argued that this practice was justified, since any shortfall that might exist in the reporters’ salaries is fully compensated by the valuable training they receive through their assignments. Which one of the following, if true about the newspaper in 1992, most seriously undermines the justification offered by the executive? (A) Senior reporters at the newspaper earned as much as reporters of similar stature who worked for the newspaper’s principle competitors. (B) Most of the newspaper’s reporters had worked there for more than ten years. (C) The circulation of the newspaper had recently reached a plateau, after it had increased steadily throughout the 1980s. (D) The union that represented reporters at the newspaper was different from the union that represented reporters at the newspaper’s competitors. (E) The newspaper was widely read throughout continental Europe and Great Britain as well as North America. 23. On a certain day, nine scheduled flights on Swift Airlines were canceled. Ordinarily, a cancellation is due to mechanical problems with the airplane scheduled for a certain flight. However, since it is unlikely that Swift would have the mechanical problems with more than one or two scheduled flights on a single day, some of the nine cancellations were probably due to something else. (A) More than one or two airplanes were scheduled for the nine canceled flights. (B) Swift Airlines has fewer mechanical problems than do other airlines of the same size. (C) Each of the canceled flights would have been longer than the average flight on Swift Airlines. (D) Swift Airlines had never before canceled more than one or two scheduled flights on a single day. (E) All of the airplanes scheduled for the canceled flights are based at the same airport. 24. The interstitial nucleus, a subregion of the brain’s hypothalamus, is typically smaller for male cats than for female cats. A neurobiologist performed autopsies on male cats who died from disease X, a disease affecting no more than .05 percent of male cats, and found that these male cats had interstitial nuclei that were as large as those generally found in female cats. Thus, the size of the interstitial nucleus determines whether or not male cats can contract disease X. Which one of the following statements, if true, most seriously weakens the argument? (A) No female cats have been known to contract disease X, which is a subtype of disease Y. (B) Many male cats who contract disease X also contract disease Z, the cause of which is unknown. (C) the interstitial nuclei of female cats who contact disease X are larger than those of female cats who do not contract disease X. (D) Of 1,000 autopsies on male cats who did not contract disease X, 5 revealed interstitial nuclei larger than those of the average male cat. (E) The hypothalamus is known not to be causally linked to disease Y, and disease X is a subtype of disease Y. 25. There should be a greater use of gasohol. Gasohol is a mixture of alcohol and gasoline, and has a higher octane rating and fewer carbon monoxide emissions than straight gasoline. Burning gasohol adds no more carbon dioxide to the atmosphere than plants remove by photosynthesis. Each of the following, if true, strengthens the argument above EXCEPT: (A) Cars run less well on gasoline than they do on gasohol. (B) Since less gasoline is needed with the use of gasohol, an energy shortage is less likely. (C) Cars burn on the average slightly more gasohol per kilometer than they do gasoline. (D) Gasohol is cheaper to produce and hence costs less at the pump than gasoline. (E) Burning gasoline adds more carbon dioxide to the atmosphere than plants can remove. 26. Barnes: The two newest employees at this company have salaries that are too high for the simple tasks normally assigned to new employees and duties that are too complex for inexperienced workers. Hence, the salaries and the complexity of the duties of these two newest employees should be reduced. Which one of the following is an assumption on which Barnes’s argument depends? (A) The duties of the two newest employees are not less complex than any others in the company. (B) It is because of the complex duties assigned that the two newest employees are being paid more than is usually paid to newly hired employees. (C) The two newest employees are not experienced at their occupations. (D) Barnes was not hired at a higher-than-average starting salary. (E) The salaries of the two newest are no higher than the salaries that other companies pay for workers with a similar level of experience. 27. These days, drug companies and health professionals alike are focusing their attention on cholesterol in the blood. The more cholesterol we have in our blood, the higher the risk that we shall die of a heart attack. The issue is pertinent since heart disease kills more North Americans every year than any other single cause. At least three factors---smoking, drinking, and exercise—can each influence levels of the cholesterol in the blood. Which one of the following can be properly concluded from the passage? (A) If a person has low blood cholesterol, then that person’s risk of fatal heart disease is low. (B) Smoking in moderation can entail as great a risk of fatal heart disease as does heavy smoking. (C) A high-cholesterol diet is the principal cause of death in North America. (D) The only way that smoking increases one’s risk of fatal heart disease is by influencing the levels of cholesterol in the blood. (E) The risk of fatal heart disease can be altered by certain changes in lifestyle. 28. Ordinary mountain sickness, a common condition among mountain climbers, and one from which most people can recover, is caused by the characteristic shortage of oxygen in the atmosphere at high altitudes. Cerebral edema, a rarer disruption of blood circulation in the brain that quickly becomes life-threatening if not correctly treated from its onset, can also be caused by a shortage of oxygen. Since the symptoms of cerebral edema resemble those of ordinary mountain sickness, cerebral edema is especially dangerous at high altitudes. Which one of the following is an assumption on which the argument depends? (A) The treatment for ordinary mountain sickness differs from the treatment for cerebral edema. (B) Cerebral edema can cause those who suffer from it to slip into a coma within a few hours. (C) Unlike cerebral edema, ordinary mountain sickness involves no disruption of blood circulation in the brain. (D) Shortage of oxygen at extremely high altitude is likely to affect thinking processes and cause errors of judgment.(E) Most people who suffer from ordinary mountain sickness recover without any special treatment. 29. The price of a full-fare coach ticket from Toronto to Dallas on Breezeway Airlines is the same today as it was a year ago, if inflation is taken into account by calculating prices in constant dollars. However, today 90 percent of the Toronto-to-Dallas coach tickets that Breezeway sells are discount tickets and only 10 percent are full-fare tickets, whereas a year ago half were discount tickets and half were full-fare tickets. Therefore, on average, people pay less today in constant dollars for a Breezeway Toronto-to-Dallas coach ticket than they did a year ago. Which one of the following, if assumed, would allow the conclusion above to be properly drawn? (A) A Toronto-to-Dallas full-fare coach ticket on Breezeway Airlines provides ticket-holders with a lower level of service today than such a ticket provided a year ago. (B) A Toronto-to-Dallas discount coach ticket on Breezeway Airlines costs about the same in constant dollars as they did a year ago. (C) All full-fare coach tickets on Breezeway Airlines cost the same in constant dollars as they did a year ago. (D) The average number of coach passengers per flight that Breezeway Airlines carries from Toronto to Dallas today is higher than the average number per flight a year ago. (E) The criteria that Breezeway Airlines uses for permitting passengers to buy discount coach tickets on the Toronto-to-Dallas route are different today than they were a year ago. 30. Combustion of gasoline in automobile engines produces benzene, a known carcinogen. Environmentalists propose replacing gasoline with methanol, which does not produce significant quantities of benzene when burned. However, combustion of methanol produces formaldehyde, also a know carcinogen. Therefore the environmentalists’ proposal has little merit. Which one of the following, if true, most supports the environmentalist’ proposal? (A) The engines of some automobiles now on the road burn diesel fuel rather than gasoline. (B) Several large research efforts are under way to formulate cleaner-burning types of gasoline. (C) In some regions, the local economy is largely dependent on industries devoted to the production and distribution of automobile fuel. (D) formaldehyde is a less potent carcinogen than benzene. (E) Since methanol is water soluble, methanol spills are more damaging to the environment than gasoline spills. Questions 31-32 Political opinion and analysis outside the mainstream rarely are found on television talk shows, and it might be thought that this state of affairs is a product of the political agenda of the television stations themselves. In fact, television stations are driven by the same economic forces as sellers of more tangible goods. Because they must attempt to capture the largest possible share of the television audience for their shows, they air only those shows that will appeal to large numbers of people. As a result, political opinions and analyses aired on television talk shows are typically bland and innocuous. 31. An assumption made in the explanation offered by the author of the passage is that (A) most television viewers cannot agree on which elements of a particular opinion or analysis are most disturbing. (B) there are television viewers who might refuse to watch television talk shows that they knew would be controversial and disturbing. (C) each television viewer holds some opinion that is outside the political mainstream, but those opinions are not the same for everyone. (D) there are television shows on which economic forces have an even greater impact than they do on television talk shows. (E) the television talk shows of different stations resemble one another in most respects. 32. The explanation offered by the author of the passage makes the assumption that (A) television station executives usually lack a political agenda of their own (B) bland and innocuous political opinions and analyses are generally in the mainstream (C) political analysts outside the mainstream are relatively indifferent to the effect their analyses have on television viewers (D) most television viewers are prepared to argue against allowing the expression of political opinions and analyses with which they disagree (E) the political opinions of television station executives are not often reflected in the television shows their stations produce Questions 33-34 Conservationist: The population of a certain wildflower is so small that the species is headed for extinction. However, this wildflower can cross-pollinate with a closely related domesticated daisy, producing viable seeds. Such cross-pollination could result in a significant population of wildflower-daisy hybrids. The daisy should therefore be introduced into the wildflower’s range, since although the hybrid would differ markedly from the wildflower, hybridization is the only means of preventing total loss of the wildflower in its range. 33. Which one of the following principles, if valid, most helps to justify the conservationist’s reasoning? (A) It is better to take measures to preserve a valued type of organism, even if those measures are drastic, than to accept a less valuable substitute for the organism. (B) It is better to preserve a type of organism that is in danger of extinction, even if surviving organisms of that type are not vigorous, than to allow something more vigorous to replace it. (C) It is better to change a type of organism that would otherwise be lost, even if the changes are radical, than to lose it entirely. (D) It is better to destroy one of two competing types of organisms, even if both are irreplaceable, than to allow both of them to be lost. (E) It is better to protect an endangered type of organism, even if doing so has some negative effects on another type of organism, than to do nothing at all. 34. Which one of the following is an assumption on which the conservationist’s reasoning depends? (A) The wildflower currently reproduces only by forming seeds. (B) The domesticated daisy was bred from wild plants that once grew in the wildflower’s range. (C) Increasing the population of the wildflower will also expand its range. (D) Wildflower-daisy hybrids will be able to reproduce. (E) The domesticated daisy will cross-pollinate with any daisy like plant. 35. Because of increases in the price of oil and because of government policies promoting energy conservation, the use of oil to heat homes fell by 40 percent from 1970 to the present, and many homeowners switched to natural gas for heating. Because switching to natural gas involved investing in equipment, a significant switch back to oil in the near future is unlikely. The prediction that ends the passage would be most seriously called into question if it were true that in the last few years. (A) the price of natural gas to heat homes has remained constant, while the cost of equipment to heat homes with natural gas has fallen sharply. (B) the price of home heating oil has remained constant, while the cost of equipment to heat home with natural gas has risen sharply. (C) the cost of equipment to heat homes with natural gas has fallen sharply, while the price of home heating oil has fallen to 1970 levels. (D) the cost of equipment to heat homes with oil has fallen sharply, while the price of heating with oil has fallen below the price of heating with natural gas (E) the use of oil to heat homes has continued to decline, while the price of heating oil has fallen to 1970 levels 36. Sometimes when their trainer gives the hand signal for “Do something creative together” two dolphins circle a pool in tandem and then leap through the air simultaneously. On the other occasions, the same signal elicits synchronized backward swims or tail-waving. These behaviors are not simply learned responses to a given stimulus. Rather, dolphins are capable of higher cognitive functions that may include the use of language and forethought. Which one of the following, if true, most strengthens the argument? (A) Mammals have some resemblance to one another with respect to bodily function and brain structure. (B) The dolphins often exhibit complex new responses to the hand signal. (C) The dolphins are given food incentives as part of their training. (D) Dolphins do not interact with humans the way they interact with one another. (E) Some of the behaviors mentioned are exhibited by dolphins in their natural habitat Test 3 37. Editorialist: Drivers with a large number of demerit points who additionally have been convicted of a serious driving-relative offense should either be sentenced to jail or be forced to receive driver reeducation, since to do otherwise would be to allow a crime to go unpunished. Only if such drivers are likely to be made more responsible drivers should driver re-education be recommended for them. Unfortunately, it is always almost impossible to make drivers with a large number of demerit points more responsible drivers. If the editorialist’s statements are true, they provide the most support for which one of the following? (A) Drivers with a large number of demerit points who have been convicted of a serious driving-related offense should be sent to jail. (B) Driver re-education offers the best chance of making drivers with a large number of demerit points responsible drivers. (C) Driver re-education is not a harsh enough punishment for anyone convicted of a serious driving-related offense who has also (D) Driver re-education should not be recommended for those who have committed no serious (E) Drivers with a larger number of demerit points but no conviction for a serious driving-related offense should receive driver re-education rather than jail 38. Amphibian populations are declining in numbers worldwide. Not coincidentally, the earth’s ozone layer has been continuously depleted throughout the last 50 years. Atmospheric ozone blocks UV-B, a type of ultraviolet radiation that is continuously produced by the sun, and which can damage genes. Because amphibians lack hair, hide, or feathers to shield them, they are particularly vulnerable to UV-B radiation. In addition, their gelatinous eggs lack the protection of leathery or hard shells. Thus, the primary cause of the declining amphibian population is the depletion of the ozone layer. Each of the following, if true, would strengthen the argument EXCEPT: (A) Of the various types of radiation blocked by atmospheric ozone, UV-B is the only type that can damage genes. (B) Amphibian populations are declining far more rapidly than are the populations of nonamphibian species whose tissues and eggs have more natural protection from UV-B. (C) Atmospheric ozone has been significantly depleted above all the areas of the world in which amphibian populations are declining. (D) The natural habitat of amphibians has not become smaller over the past century. (E) Amphibian populations have declined continuously for the last 50 years. 39. Quasars---celestial objects so far away that their light takes at least 500 million years to reach Earth---have been seen since 1963. For anything that far away to appear from Earth the way quasars do, it would have to burn steadily at a rate that produces more light than 90 billion suns would produce. But nothing that burns at a rate that produces that much light could exist for more than about 100 million years. If the statements above are true, which one of the following must also be true on the basis of them? (A) Instruments in use before 1963 were not sensitive enough to permit quasars to be seen. (B) Light from quasars first began reaching Earth in 1963. (C) Anything that from Earth appears as bright as a quasar does must produce more light than would be produced by 90 billion suns. (D) Nothing that is as far from Earth as quasars are can continue to exist for more than about 100 million years. (E) No quasar that has ever been seen from Earth exists any longer. 40. More and more computer programs that provide solutions to mathematical problems in engineering are being produced, and it is thus increasingly unnecessary for practicing engineers to have thorough understanding of fundamental mathematical principles. Consequently, in training engineers who will work in industry, less emphasis should be placed on mathematical principles, so that space in the engineering curriculum will be available for other important subjects. Which one of the following, if true, most seriously weakens the argument given for the recommendation above? (A) The effective use of computer program that provide solutions to mathematical problems in engineering requires an understanding of mathematical principles. (B) Many of the computer programs that provide solutions to mathematical problems in engineering are already in routine use. (C) Development of composites and other such new materials has meant that the curriculum for engineers who will work in industry must allow time for teaching the properties of these materials. (D) Most of the computer programs that provide solutions to mathematical problems in engineering can be run on the types of computers available to most engineering firms. (E) The engineering curriculum already requires that engineering students be familiar with and able to use a variety of computer programs. 41. Raymond Burr played the role of lawyer Perry Mason on television. Burr’s death in 1993 prompted a prominent lawyer to say “Although not a lawyer, Mr. Burr strove for such authenticity that we feel as if we lost one of our own.” This comment from a prestigious attorney provides appalling evidence that, in the face of television, even some legal professionals are losing their ability to distinguish fiction from reality. The reasoning in the argument is flawed because the argument (A) takes the views of one lawyer to represent the views of all lawyers (B) criticizes the lawyer rather than the lawyer’s statement (C) presumes that the lawyer is qualified to evaluate the performance of an actor (D) focuses on a famous actor’s portrayal of a lawyer rather than on the usual way in which lawyers are portrayed on television (E) ignores the part of the lawyer’s remark that indicates an awareness of the difference between reality and fiction 42. For next year, the Chef’s Union has requested a 10 percent salary increase for each of its members, whereas the Hotel Managers’ Union has requested only an 8 percent salary increase for each of its members. These facts demonstrate that the average dollar amount of the raises that the Chefs’ Union has requested for next year is greater than that of the raises requested by the Hotel Managers’ Union. Which one of the following, if true, most strengthens the argument? (A) The Chefs’ Union has many more members than does the Hotel Managers’ Union. (B) The Chefs’ Union is a more powerful union than is the Hotel Managers’ Union and is therefore more likely to obtain the salary increases it requests (C) The current salaries of the members of the Chefs’ Union are, on average, higher than the current salaries of the members of the Hotel Managers’ Union (D) The average dollar amount of the raises that the members of the Chefs’ Union received last ear was equal to the average dollar amount of the raises that the members of the Hotel Managers’ Union received (E) The members of the Chefs’ Union received salary increases of 10 percent in each of the last two years, while the members of the Hotel Managers’ Union received salary increases of only 8 percent in each of the last two years 43. Zoos have served both as educational resources and as entertainment. Unfortunately, removing animals from their natural habitats to stock the earliest zoos reduced certain species’ populations, endangering their survival. Today most new zoo animals are obtained from captive breeding programs, and many zoos now maintain breeding stocks for continued propagation of various species. This makes possible efforts to reestablish endangered species in the wild. Which one of the following statements is most strongly supported by the information above? (A) Zoos have played an essential role in educating the public about endangered species. (B) Some specimens of endangered species are born and bred in zoos. (C) No zoos exploit wild animals or endanger the survival of species. (D) Nearly all of the animals in zoos today were born in captivity. (E) The main purpose of zoos has shifted from entertainment to education. 44. Legal theorist: It is unreasonable to incarcerate anyone for any other reason than that he or she is a serious threat to the property or lives of other people. The breaking of a law does not justify incarceration, for lawbreaking proceeds either from ignorance of the law or of the effects of one’s actions, or from the free choice of the lawbreaker. Obviously mere ignorance cannot justify incarcerating a lawbreaker, and even free choice on the part of the lawbreaker fails to justify incarceration, for free choice proceeds from the desires of an agent, and the desires of an agent are products of genetics and environmental conditioning, neither of which is controlled by the agent The claim in the first sentence of the passage plays which one of the following roles in the argument (A) It is offered as a premise that helps to show that no actions are under the control of the agent (B) It is offered as background information necessary to understand the argument (C) It is offered as the main conclusion that the argument is designed to establish (D) It is offered as evidence for the stated claim that protection of life and property is more important than retribution for past illegal acts (E) It is offered as evidence for the stated claim that lawbreaking proceeds from either ignorance of the law, or ignorance of the effects of one’s actions, or free choice 45. Commissioner: Budget forecasters project a revenue shortfall of a billion dollars in the coming fiscal year. Since there is no feasible way to increase the available funds, our only choice is to decrease expenditures. The plan before you outlines feasible cuts that would yield savings of a billion dollars over the coming fiscal year. We will be able to solve the problem we face, therefore, only if we adopt this plan. This reasoning in the commissioner’s argument is flawed because this argument (A) relies on information that is far from certain (B) confuses being an adequate solution with being a required solution (C) inappropriately relies on the opinions of experts (D) inappropriately employs language that is vague (E) takes for granted that there is no way to increase available funds 46. Twelve healthy volunteers with the Apo-A-IV-1 gene and twelve healthy volunteers who instead have the Apo-A-IV-2 gene each consumed a standard diet supplemented daily by a high-cholesterol food. A high level of cholesterol in the blood is associated with an increased risk of heart disease. After three weeks, the blood cholesterol levels of the subjects in the second group were unchanged, whereas the blood cholesterol levels of those with the Apo-A-IV-1 gene rose 20 percent. Which one of the following is most strongly supported by the information above? (A) Approximately half the population carries a gene that lowers cholesterol levels. (B) Most of those at risk of heart disease may be able to reduce their risk by adopting a low-cholesterol diet. (C) The bodies of those who have the Apo-A-IV-2 gene excrete cholesterol when blood cholesterol reaches a certain level. (D) The presence of the Apo-A-IV-1 gene seems to indicate that a person has a lower risk of heart disease. (E) The presence of the Apo-A-IV-2 gene may inhibit the elevation of blood cholesterol. 47. High school students who feel that they are not succeeding in school often drop out before graduating and go to work. Last year, however, the city’s high school dropout rate was significantly lower than the previous year’s rate. This is encouraging evidence that the program instituted two years ago to improve the morale of high school students has begun to take effect to reduce dropouts. Which one of the following, if true about the last year, most seriously weakens the argument? (A) There was a recession that caused a high level or unemployment in the city. (B) The morale of students who dropped out of high school had been low even before they reached high school. (C) As in the preceding year, more high school students remained in school than dropped out. (D) High schools in the city established placement offices to assist their graduates in obtaining employment. (E) The antidropout program was primarily aimed at improving students’ morale in those high schools with the highest dropout rates. 48. The television show Henry was not widely watched until it was scheduled for Tuesday evenings immediately after That’s Life, the most popular show on television. During the year after the move, Henry was consistently one of the ten most-watched shows on television. Since Henry’s recent move to Wednesday evenings, however, it has been watched by far fewer people. We must conclude that Henry was widely watched before the move to Wednesday evenings because it followed That’s Life and not because people especially liked it. Which one of the following, if true, most strengthens the argument? (A) Henry has been on the air for three years, but That’s Life has been on the air for only two years. (B) The show that replaced Henry on Tuesdays has persistently had a low number of viewers in the Tuesday time slot. (C) The show that now follows That’s Life on Tuesdays has double the number of viewers it had before being moved. (D) After its recent move to Wednesday, Henry was aired at the same time as the second most popular show on television. (E) That’s Life was not widely watched during the first year it was aired. Questions 49-50 Joseph: My encyclopedia says that the mathematician Pierre de Fermat died in 1665 without leaving behind any written proof for a theorem that he claimed nonetheless to have proved. Probably this alleged theorem simply cannot be proved, since---as the article points out---no one else has been able to prove it. Therefore it is likely that Fermat was either lying or else mistaken when he made his claim. Laura: Your encyclopedia is out of date. Recently someone has in fact proved Fermat’s theorem. And since the theorem is provable, your claim---that Fermat was lying or mistaken---clearly is wrong. 49. Joseph’s statement that “this alleged theorem simply cannot be proved” plays which one of the following roles in his argument? (A) an assumption for which no support is offered (B) a subsidiary conclusion on which his argument’s main conclusion is based (C) a potential objection that his argument anticipates and attempts to answer before it is raised (D) the principle claim that his argument is structured to refute (E) background information that neither supports nor undermines his argument’s conclusion 50. Which one of the following most accurately describes a reasoning error in Laura’s argument? (A) It purports to establish its conclusion by making a claim that, if true, would actually contradict that conclusion. (B) It mistakenly assumes that the quality of a person’s character can legitimately be taken to guarantee the accuracy of the claims that person has made. (C) It mistakes something that is necessary for its conclusion to follow for something that ensures that the conclusion follows. (D) It uses the term “provable” without defining it. (E) It fails to distinguish between a true claim that has mistakenly between believed to be false and a false claim that has mistakenly been believed to be true. 51. People who have political power tend to see new technologies as a means of extending or protecting their power, whereas they generally see new ethical arguments and ideas as a threat to it. Therefore, technical ingenuity usually brings benefits to those who have this ingenuity, whereas ethical inventiveness brings only pain to those who have this inventiveness. Which one of the following statements, if true, most strengthens the argument? (A) Those who offer new ways of justifying current political power often reap the benefits of their own innovations. (B) Politically powerful people tend to reward those who they believe are useful to them and to punish those who they believe are a threat. (C) Ethical inventiveness and technical ingenuity are never possessed by the same individuals (D) New technologies are often used by people who strive to defeat those who currently have political power. (E) Many people who possess ethical inventiveness conceal their novel ethical arguments for fear of retribution by the politically powerful. 52. Birds need so much food energy to maintain their body temperatures that some of them spend most of their time eating. But a comparison of a bird of a seed-eating species to a bird of a nectar-eating species that has the same overall energy requirement would surely show that the seed-eating bird spends more time eating than does the nectar-eating bird, since a given amount of nectar provides more energy than does the same amount of seeds The argument relies on which one of the following questionable assumptions (A) Birds of different species do not generally have the same overall energy requirements as each other (B) The nectar-eating bird does not sometimes also eat seeds (C) The time it takes for the nectar-eating bird to eat a given amount of nectar is not longer than the time it takes the seed-eating bird to eat the same amount of seeds (D) The seed-eating bird does not have a lower body temperature than that of the nectar-eating bird (E) The overall energy requirements of a given bird do not depend on factors such as the size of the bird, its nest-building habits; and the climate of the region in which it lives 53. When investigators discovered that the director of a local charity had repeatedly overstated the number of people his charity had helped, the director accepted responsibility for the deception. However, the investigators claims that journalists were as much to blame as the director was for inflating the charity’s reputation, since they had naively accepted what the director told them, and simply reported as fact the numbers he gave them. Which one of the following principles, if valid, most helps to justify the investigators’ claim (A) Anyone who works for a charitable organization is obliged to be completely honest about the activities of that organization. (B) Anyone who knowingly aids a liar by trying to conceal the truth from others is also a liar. (C) Anyone who presents as factual a story that turns out to be untrue without first attempting to verify that story is no less responsible for the consequences of that story than anyone else is. (D) Anyone who lies in order to advance his or her own career is more deserving of blame than someone who lies in order to promote a good cause. (E) Anyone who accepts responsibility for a wrongful act that he or she committed is less deserving of blame than someone who tries to conceal his or her own wrongdoing. 54. Telephone companies are promoting “voice mail” as an alternative to the answering machine. By recording messages from callers when a subscriber does not have access to his or her telephone, voice mail provides a service similar to that of an answering machine. The companies promoting this service argue that it will soon make answering machines obsolete, since it is much more convenient, more flexible, and less expensive than an answering machine. which one of the following, if true, most calls into question the argument made by the companies promoting voice mail? (A) Unlike calls made to owners of answering machines, all telephone calls made to voice-mail subscribers are completed, even if the line called is in use at the time of the call. (B) The surge in sales of answering machines occurred shortly after they were first introduced to the electronics market. (C) Once a telephone customer decides to subscribe to voice mail, that customer can cancel the service at any time. (D) Answering machines enable the customer to hear who is calling before the customer decides whether to answer the telephone, a service voice mail does not provide. (E) The number of messages a telephone answering machine can record is limited by the length of the magnetic tape on which calls are recorded Test 4 Questions 55-56 The simple facts are these: the number of people killed each year by bears is about the same as the number of people killed by lightning of golf courses. And the number of people killed by lightning on golf courses each year is about the same as the number of people electrocuted by electric blenders. All the horrible myths and gruesome stories aside, therefore a grizzly bear is in fact about as dangerous as an electric blender or a game of golf. 55. Which one of the following is an assumption that the author relies upon in the passage? (A) Most incidents involving grizzly bears are fatal. (B) Grizzly bears are no longer the danger they once were. (C) The number of fatalities per year is an adequate indication of something’s dangerousness. (D) A golf course is a particularly dangerous place to be in a thunderstorm. (E) Something is dangerous only if it results in death in the majority of cases. 56. Which one of the following, if true, would most effectively undermine the author’s argument? (A) Although the number of people killed by lightning on golf courses each year is very small, the total number of lightning fatalities is many times greater. (B) Electric blenders are among the safest housed hold appliances; were the author to compare fatalities from electrical appliances in general, she would get a much higher figure. (C) Most people would rather take their chances with benders and golf games than with grizzly bears. (D) Bears in general---including black, brown, and cinnamon bears, as well as grizzly bears---kill many more people than do electric blenders. (E) Statistics show that the number of times people use electric blenders each year exceeds the number of times people play golf each year, which in turn far exceeds the number of contacts people have with grizzly bears each year. 57. Free public education is the best form of education there is. Therefore, we must fight to ensure its continued existence; that is, we must be ready to defend the principle of equality of educational opportunity. Because this principle is well worth defending, it is clear that free public education is better than any other form of education. Which one of the following illustrates the same weak reasoning as found in the passage? (A) I love music, and that’s why I listen to it constantly. I have my stereo or radio on every waking minute. Since I lay music all the time, I must really love it. (B) Books are my most valuable possessions. My books are like my friends---each pleases me in different ways. Just as I would give up everything to save my friends, so too with my books. (C) I would much rather be poor and respected than be rich and despises. To have the respect of others is far more valuable than to have millions of dollars. (D) I have never been betrayed by any of my friends. They have been true to me through good times and bad. Therefore I will never betray any of my friends. (E) Because every plant I have ever seen has green leaves, I have concluded that all plants must have green leaves. This looks like a plant but it does not have green leaves, so it cannot be a plant. 58. Some people say that the scarcity of food is a function of the finite limits of the earth’s resources, coupled with a relentless rate of population growth. This analysis fails to recognize, however, that much of the world’s agricultural resources are used to feed livestock instead of people. In the United States, for example, almost one-half of the agricultural acreage is devoted to crops fed to livestock. A steer reduces twenty-one pounds of inexpensive grain to one pound of expensive meat. Thus, the scarcity of food is not merely a function of limited resources and population growth. Which one of the following is an assumption that would allow the conclusion in the argument to be properly drawn? (A) People prefer eating meat to eating grain. (B) Meat is twenty-one times more expensive than grain. (C) The limits of the earth’s agricultural resources are not finite. (D) More than one-half of the agricultural acreage in the United States is devoted to drops fed to humans. (E) Growing crops for human consumption on the acreage currently devoted to crops for livestock will yield more food for more people. 59. Top college graduates are having more difficulty demonstrating their superiority to prospective employers than did the top students of twenty years ago when an honors degree was distinction enough. Today’s employers are less impressed with the honors degree. Twenty years ago no more than 10 percent of a given class graduated with honors. Today, however, because of grade inflation, the honors degree goes to more than 50 percent of a graduating class. Therefore, to restore confidence in the degrees they award, colleges must take steps to control grade inflation. Which one of the following is an assumption that, if true, would support the conclusion in the passage? (A) Today’s students are not higher achievers than the students of twenty years ago. (B) Awarding too many honors degrees causes colleges to inflate grades. (C) Today’s employers rely on honors ranking in making their hiring decisions. (D) It is not easy for students with low grades to obtain jobs. (E) Colleges must make employers aware of the criteria used to determine who receives an honor degree. 60. One of the more reliable methods of determining regional climatic conditions in prehistoric periods is to examine plant pollen trapped in glacial ice during ancient times. By comparing such pollen samples with spores take from modern vegetation, scientists can figure out approximately what the weather was like at the time of pollen deposition. Furthermore, by submitting the prehistoric samples to radiocarbon dating techniques, we can also determine when certain climatic conditions were prevalent in that portion of the globe. Which one of the following may be inferred from the information in the passage? (A) The earth has undergone several glacial periods. (B) Radiocarbon dating can be corroborated by glacial evidence. (C) Similarities between prehistoric and contemporary climates do not exist. (D) Pollen deposition is a fairly continuous process. (E) Certain flora are reliably associated with particular climatic conditions. 61. Investigators concluded that human failure was not responsible for the fatal airplane crash last August, and since that time new and more stringent rules for identifying and reporting mechanical problems have been in effect. That accounts for the fact that reports of airplane mechanical problems have increased in frequency by 50 percent since last August. Which one of the following is an assumption underlying the argument in the passage? (A) Airplane travel is still relatively safe, despite the increase in reported mechanical problems. (B) Mechanical problems in airplanes have increased dramatically since last August. (C) Mechanical problems in airplanes have not increased by 50 percent since last August. (D) Airlines are less reluctant to report mechanical problems than they previously were. (E) Mechanical problems in airplanes have become easier to detect since last August. 62. Brushing your teeth regularly no matter which toothpaste you use, will reduce your chances of tooth decay. Scientists have concluded that, when you brush, you reduce tooth decay by removing the film of plaque that forms on teeth and gums. So, you can forget about fluorides: brush your teeth carefully and say goodbye to cavities. Which one of the following is a criticism of the reasoning in the argument? (A) Brushing with fluoride toothpaste has been shown to reduce tooth decay. (B) The fact that brushing will reduce tooth decay does not show that fluorides are of no value. (C) Few people adequately remove plaque by brushing. (D) People have plaque on their teeth most of the time. (E) Scientists have been wrong about fluorides. 63. Some good cooks are gourmet cooks who pride themselves on always using extravagantly rich ingredients in elaborate recipes. Some good cooks can be characterized as fast-food cooks. They may use rich ingredients as long as the recipes are easy to follow and take little time. Other good cooks are health food enthusiasts, who are concerned primarily with the nutritional value of food. But even though not all good cooks are big eaters, they all enjoy preparing and serving food. If the information in the passage is true, which one of the following CANNOT be true? (A) Most good cooks do not use extravagantly rich ingredients. (B) Everyone who enjoys preparing and serving food is a good cook. (C) More good cooks who use extravagantly rich ingredients are big eaters than are good cooks who do not use such ingredients. (D) There are fewer good cooks who enjoy serving and preparing food than there are good cooks who are big eaters. (E) Gourmet cooks, fast-food cooks, and cooks who are health food enthusiasts are all big eaters. Questions 64-65 If the city council institutes new parking regulations, city revenues will surely increase, since studies have conclusively shown that, if such parking regulations are put into effect, there is an increase in parking violations, and an increase in parking violations will result in a greater number of parking fines collected. 64. If the statements in the passage are true, which one of the following must also be true? (A) Unless there is an increase in the number of parking violations in the city, city revenues will not increase. (B) If the city council institutes new parking regulations, the council will fall from favor with the citizens. (C) The city council will institute new parking regulations only if an increase in city revenues can be expected to result. (D) If the city council’s new regulations cause more parking violators to be ticketed, the city revenues will increase. (E) Unless the city institutes a complex system of parking regulations, the city cannot expect traffic violations to increase. 65. According to advertisements, the higher a suntan lotion’s sun protection factor, or SPE, the more protection from sunburn. In order for a suntan lotion to work, however, one has to remember to put it on before going in the sun, put on an adequate amount to cover the skin, and reapply it as needed. Therefore, it really does not matter what SPE a suntan lotion has. Which one of the following best identifies the error in reasoning made in the passage? (A) It is unreasonable to assume that the only purpose of a suntan lotion is to provide protection from sunburn. (B) Because some people get sunburned more easily than other, the fact that there are different SPEs cannot be ignored. (C) It cannot be concluded that the SPEs is not important just because there are requirements for the application of the suntan lotion. (D) It is unreasonable to assume that all suntan lotions required the same application procedures in order to work effectively. (E) There is no reason to assume that manufacturers are unaware that people sometimes forget to apply suntan lotion before going in the sun. 66. A recent survey showed that many workers in a certain company are dissatisfied with their jobs. The survey also showed that most of the dissatisfied workers believe that they have little control over their job assignments. Therefore, to increase workers’ job satisfaction the company’s management need only concentrated on changing workers’ beliefs regarding the degree of control they have over their job assignment. Which one of the following, if also shown by the survey, would most seriously call into question the conclusion made by the author of the passage? (A) The dissatisfied workers feel that their wages are too low and working conditions are unsatisfactory. (B) The number of workers in the company who are satisfied with their jobs is greater than the number of workers who are dissatisfied. (C) The workers in the company are more dissatisfied than workers in other companies. (D) Most people in company management believe that the workers already have too much control over their work. (E) The workers in the company who are satisfied with their jobs believe that they have a lot of control over their job assignments. 67. It has always been difficult to understand the basis of politics in the People’s Republic of China. Because the system is effectively closed, it is impossible to know with any degree of confidence who is allied with whom and for what reasons. Yet Chinese politics does exhibit many of the external characteristics of factional political systems, as found in more open societies. It is legitimate to conclude, therefore, that China has a factional political system. Which one of the following, if true, would confirm the author’s conclusion that China has a factional political system? (A) All open political systems are factional political systems. (B) All factional political systems are closed political systems. (C) All closed political systems are factional political systems. (D) China’s political system is more open than many existing factional political systems. (E) China’s political system is more closed than all existing factional political systems. 68. Since no one returns from death, we can never be certain about what passes through the mind of the dying person. For the unconscious, the confused, and the heavily sedated, these final moments are probably meaningless. However, for the mentally alert, it is quite possible that death presents itself as an unbelievably glorious experience, a flight into an entirely new universe of sensation. Why should we think so? Some people who have been reprieved from “certain” death at the last moment have experienced what goes through the consciousness of those who are not so fortunate. For example, parachutists who have survived falls report experiences that resemble psychedelic “trips”. The primary point of the argument in the passage is (A) no one returns from death (B) dying can be a glorious experience (C) we can never know what passes through the mind of a dying person (D) some people are reprieved from death at the last moment (E) some people “die”, yet live to report their experiences 69. History textbooks frequently need to be revised. The reasons for this are clear: new discoveries of documents and remains, the discovery of mistaken inferences in prior histories, the discovery of previously unnoticed relationships among data, and the application of hitherto undiscovered principles of natural science all may indicate inadequacies in current history texts. Any of these considerations may require that the past be reinterpreted in a manner that is new and more illuminating. Which one of the following can be inferred from the argument in the passage? (A) The interpretation of historical events is affected by natural science. (B) The past is constantly renewed because of illuminating reinterpretations. (C) History books are outdated as soon as they are written. (D) Natural scientists also function as historians. (E) Historians’ mistaken inferences are caused by unnoticed relationships among data. 70. A low-pressure weather system is approaching Plainville; rainfall results from about 70 percent of such systems in the Plainville area. Moreover, the current season, spring, is the time of year in which thundershowers, which sometimes result from low-pressure systems, are most likely to occur in Plainville. Knowing which one of the following, in addition to the information above, would be most useful for determine the probability that Plainville will have a thundershower soon? (A) the percentage of thundershowers in Plainville that occur in the spring (B) the percentage of spring rainfalls in Plainville that are thundershowers (C) the percentage of thundershowers in Plainville that result from low-pressure systems (D) whether low-pressure systems in other areas are associated with rainfall (E) whether Plainville has more or fewer thundershowers each spring than do near by towns 71. More than a year ago, the city announced that police would crack down on illegally parked cars and that resources would be diverted from writing speeding tickets to ticketing illegally parked cars. But no crackdown has taken place. The police chief claims that resources have had to be diverted from writing speeding tickets to combating the city’s staggering drug problem. Yet the police are still writing as many speeding tickets as ever. Therefore, the excuse about resources being tied up in fighting drug-related crime simply is not true. The conclusion in the passage depends on the assumption that (A) ever member of the police force is qualified to work on combating the city’s drug problem (B) drug-related crime is not as serious a problem for the city as the police chief claims it is (C) writing speeding tickets should be as important a priority for the city as combating drug-related crime (D) the police could be cracking down on illegally parked cars and combating the drug problem without having to reduce writing speeding tickets (E) the police cannot continue writing as many speeding tickets as ever while diverting resources to combating drug-related crime 72. A gas tax of one cent per gallon would raise one billion dollars per year at current consumption rates. Since a tax of fifty cents per gallon would therefore raise fifty billion dollars per year, it seems a perfect way to deal with the federal budget deficit. This tax would have the additional advantage that the resulting drop in the demand for gasoline would be ecologically sound and would keep our county from being too dependent on foreign oil producers. Which one of the following most clearly identifies an error in the author’s reasoning? (A) The author cites irrelevant data. (B) The author relies on incorrect current consumption figures. (C) The author makes incompatible assumptions. (D) The author mistakes an effect for a cause. (E) The author appeals to conscience rather than reason. Test 5 73. There is no reason why the work of scientists has to be officially confirmed before being published. There is a system in place for the confirmation or disconfirmation of scientific findings, namely, the replication of results by other scientists. Poor scientific work on the part of any one scientists, which can include anything from careless reporting practices to fraud, is not harmful. It will be exposed and rendered harmless when other scientists conduct experiments and obtain disconfirmatory results. Which one of the following, if true, would weaken the argument? (A) Scientific experiments can go unchallenged for many years before they are replicated. (B) Most scientists work in universities, where their work is submitted to peer review before publication. (C) Most scientists are under pressure to make their work accessible to the scrutiny of replication. (D) In scientific experiments, careless reporting is more common than fraud. (E) Most scientists work as part of a team rather than alone. 74. Governments have only one response to public criticism of socially necessary services: regulation of the activity of providing those services. But governments inevitably make the activity more expensive by regulating it, and that is particularly troublesome in these times of strained financial resources. However, since public criticism of childcare services has undermined all confidence in such services, and since such services are socially necessary, the government is certain to respond. Which one of the following statements can be inferred from the passage? (A) The quality of child-care will improve. (B) The cost of providing child-care services will increase. (C) The government will use funding to foster advances in child-care. (D) If public criticism of policy is strongly voiced, the government is certain to respond. (E) If child-care services are not regulated, the cost of providing childcare will not increase. 75. Advertisers are often criticized for their unscrupulous manipulation of people’s tastes and wants. There is evidence, however, that some advertisers are motivated by moral as well as financial considerations. A particular publication decided to change its image from being a family newspaper to concentrating one sex and violence, thus appealing to a different readership. Some advertisers withdrew their advertisements from the publication, and this must have been because they morally disapproved of publishing salacious material. Which one of the following, if true, would most strengthen the argument? (A) The advertisers switched their advertisements to other family newspapers. (B) Some advertisers switched from family newspapers to advertise in the changed publication. (C) The advertisers expected their product sales to increase if they stayed with the changed publication, but to decrease if they withdrew. (D) People who generally read family newspapers are not likely to buy newspapers that concentrate on sex and violence. (E) It was expected that the changed publication would appeal principally to those in a different income group. 76. Learning how to build a nest plays an important part in the breeding success of birds. For example, Dr. Snow has recorded the success of a number of blackbirds in several successive years. He finds that birds nesting for the first time are less successful in breeding than are older birds, and also less successful than they themselves are a year later. This cannot be a mere matter of size and strength, since blackbirds, like the great majority of birds, are fully grown when they leave the nest. It is difficult to avoid the conclusion that they benefit by their nesting experience. Which one of the following, if true, would most weaken the argument? (A) Blackbirds build better nests than other birds. (B) The capacity of blackbirds to lay viable eggs increase with each successive trial during the first few years of reproduction. (C) The breeding success of birds nesting for the second time is greater than that of birds nesting for the first time. (D) Smaller and weaker blackbirds breed just as successfully as bigger and stronger blackbirds. (E) Up to 25 percent of all birds are killed by predators before they start to nest. 77. How do the airlines expect to prevent commercial plane crashes? Studies have shown that pilot error contributes to two-thirds of all such crashes. To address this problem, the airline have upgraded their training programs by increasing the hours of classroom instruction and emphasizing communication skills in the cockpit. But it is unrealistic to expect such measures to compensate for pilots’ lack of actual flying time. Therefore, the airlines should rethink their training approach to reducing commercial crashes. Which one of the following is an assumption upon which the argument depends? (A) Training programs can eliminate pilot errors. (B) Commercial pilot routinely undergo additional training throughout their careers. (C) The number of airline crashes will decrease if pilot training programs focus on increasing actual flying time. (D) Lack of actual flying time is an important contributor to pilot error in commercial plane crashes. (E) Communication skills are not important to pilot training programs. Questions 78-79 Despite improvements in treatment for asthma, the death rate from this disease has doubled during the past decade from its previous rate. Two possible explanations for this increase have been offered. First, the recording of deaths due to asthma has become more widespread and accurate in the past decade than it had been previously. Second, there has been an increase in urban pollution. However, since the rate of deaths due to asthma has increased dramatically even in cities with long-standing, comprehensive medical records and with little or no urban pollution, one must instead conclude that the cause of increased deaths is the use of bronchial inhalers by asthma sufferers to relieve their symptoms. 78. Each of the following, if true, provides support to the argument EXCEPT: (A) Urban populations have doubled in the past decade. (B) Records of asthma deaths are as accurate for the past twenty years as for the past ten years. (C) Evidence suggests that inhalers make the lungs more sensitive to irritation by airborne pollen. (D) By temporarily relieving the symptoms of asthma, inhalers encourage sufferers to avoid more beneficial measures. (E) Ten years ago bronchial inhalers were not available as an asthma treatment. 79. Which one of the following is an assumption on which the argument depends? (A) Urban pollution has not doubled in the past decade. (B) Doctors and patients generally ignore the role of allergies in asthma. (C) Bronchial inhalers are unsafe, even when used according to the recommended instructions. (D) The use of bronchial inhalers aggravates other diseases that frequently occur among asthma sufferers and that often lead to fatal outcomes even when the asthma itself does not. (E) Increased urban pollution, improved recording of asthma deaths, and the used of asthma deaths, and the use of bronchial inhalers are the only possible explanations of the increased death rate due to asthma. 80. Advertisement: Clark brand-name parts are made for cars manufactured in this country. They satisfy all of our government automotive tests-the toughest such tests in the world. With foreign-made parts, you never know which might be reliable and which are cheap look-alikes that are poorly constructed and liable to cost you hundreds of dollars in repairs. Therefore, be smart and insist on brand-name parts by Clark for your car. The argument requires the assumption that (A) Clark parts are available only in this country (B) foreign-made arts are not suitable for cars manufactured in this country (C) no foreign-made parts satisfy our government standards (D) parts that satisfy or government standards are not as poorly constructed as cheap foreign made parts (E) if parts are made for cars manufactured in our country, they are not poorly constructed 81. Nuclear fusion is a process whereby the nuclei of atoms are joined, or “fused”, and in which energy is released. One of the by-products of fusion is helium-4 gas. A recent fusion experiment was conducted using “heavy” water contained in a sealed flask. The flask was, in turn, contained in an air-filled chamber designed to eliminate extraneous vibration. After the experiment, a measurable amount of helium-4 gas was found in the air of the chamber. The experiments cited this evidence in support of their conclusion that fusion had been achieved. Which one of the following, if true, would cast doubt on the experimenters’ conclusion? (A) Helium-4 was not the only gas founding the experiment chamber. (B) When fusion is achieved, it normally produces several by-products, including tritium and gamma rays. (C) The amount of helium-4 found in the chamber’s air did not exceed the amount of helium-4 that is found in ordinary air. (D) Helium-4 gas rapidly breaks down, forming ordinary helium gas after a few hours. (E) Nuclear fusion reactions are characterized by the release of large amount of heat. 82. Every photograph, because it involves the light rays that something emits hitting film, must in some obvious sense be true. But because it could always have been made to show things differently than it does, it cannot express the whole truth and, in that sense, is false. Therefore, nothing can ever be definitively proved with a photograph. Which one of the following, if true, would cast doubt on the experimenters’ conclusion? (A) Helium-4 was not the only gas found in the experiment chamber. (B) When fusion is achieved, it normally produces several by-products, including tritium and gamma rays. (C) The amount of helium-4 found in the chamber’s air did not exceed the amount of helium-4 that is found in ordinary air. (D) Helium-4 gas rapidly breaks down, forming ordinary helium gas after a few hours. (E) Nuclear fusion reactions are characterized by the release of large amount of heat. Questions 83-84 Some cleaning fluids, synthetic carpets, wall paneling, and other products release toxins, such as formaldehyde and benzene, into the household air supply. This is not a problem in well-ventilated houses, but it is a problem in houses that are so well insulated that they trap toxins as well as heat. Recent tests, however, demonstrate that houseplants remove some household toxins from the air and thereby eliminate their danger. In one test, 20 large plants eliminated formaldehyde from a small, well-insulated house. 83. Assume that a person who lives in a small, well-insulated house that contains toxin-releasing products places houseplants, such as those tested, in the house. Which one of the following can be expected as a result? (A) There will no longer be any need to ventilate the house. (B) The concentration of toxins in the household air supply will remain the same. (C) The house will be warm and have a sage air supply. (D) If there is formaldehyde in the household air supply, its level will decrease. (E) If formaldehyde benzene are being released of each will decrease. 84. The passage is structured to lead to which one of the following conclusions? (A) Houseplants can remove benzene from the air. (B) Nonsynthetic products do not release toxins into houses. (C) Keeping houseplants is an effective means of trapping heat in a poorly insulated house. (D) Keeping houseplants can compensate for some of the negative effects of poor ventilation. (E) The air in a well-insulated house with houseplants will contain fewer toxins than the air in a well-ventilated house without houseplants. Questions 85-86 In opposing the 1970 Clean Air Act, the United States automobile industry argued that meeting the act’s standards for automobile emissions was neither economically feasible nor environmentally necessary. However, the catalytic converter, invented in 1967, enabled automakers to meet the 1970 standards efficiently. Currently, automakers are lobbying against the government’s attempt to pass legislation that would tighten restrictions on automobile emissions. The automakers contend that these new restrictions would be overly expensive and unnecessary to efforts to curb air pollution. Clearly, the automobile industry’s position should not be heeded. 85. Which one of the following most accurately expresses the method used to counter the automakers’ current position? (A) The automakers’ premises are shown to lead to a contradiction. (B) Facts are mentioned that show that the automakers are relying on false information. (C) A flaw is pointed out in the reasoning used by the automakers to reach their conclusion. (D) A comparison is drawn between the automakers’ current position and a position they held in the past. (E) Evidence is provided that the new emissions legislation is both economically feasible and environmentally necessary. 86. Which one of the following, if true, lends the most support to the automakers’ current position? (A) The more stringent the legislation restricting emission becomes, the more difficult it becomes for automakers to provide the required technology economically. (B) Emissions-restriction technology can often be engineered so as to avoid reducing the efficiency with which an automobile uses fuel. (C) Not every new piece of legislation restricting emission requires new automotive technology in order for automakers to comply with it. (D) The more automobiles there are on the road, the more stringent emission restrictions must be to prevent increased overall air pollution. (E) Unless forced to do so by the government, automakers rarely make changes in automotive technology that is not related to profitability. 87. A survey was recently conducted among ferry passengers on the North Sea. Among the results was this: more of those who had taken anti-seasickness medication before their trip reported symptoms of seasickness than those who had not taken such medication. It is clear, then, that despite claims by drug companies that clinical tests show the contrary, people would be better off not taking anti-seasickness medications. Which one of the following, if true, would most weaken the conclusion above? (A) Given rough enough weather, most ferry passengers will have some symptoms of seasickness. (B) The clinical tests reported by the drug companies were conducted by the drug companies’ staffs. (C) People who do not take anti-seasickness medication are just as likely to respond to a survey on seasickness as people who do. (D) The seasickness symptoms of the people who took anti-seasickness medication would have been more severe had they not taken the medication. (E) People who have spent money on anti-seasickness medication are less likely to admit symptoms of seasickness than those who have not. 88. Court records from medieval France show that in the years 1300 to 1400 the number of people arrested in the French realm for “violent interpersonal crimes” (not committed in wars) increased by 30 percent over the number of people arrested for such crimes in the years 1200 to 1300. If the increase was not the result of false arrests, therefore, medieval France had a higher level of documented interpersonal violence in the years 1300 to 1400 than in the years 1200 to 1300. Which one of the following statements, if true, most seriously weakens the argument? (A) In the years 1300 to 1400 the French government’s category of violent crimes included an increasing variety of interpersonal crimes that are actually nonviolent. (B) Historical accounts by monastic chroniclers in the years 1300 to 1400 are filled with descriptions of violent attacks committed by people living in the French realm. (C) The number of individual agreements between two people in which they swore oaths not to attack each other increased substantially after 1300. (D) When English armies tried to conquer parts of France in the mid-to late 1300s, violence in the northern province of Normandy and the southwestern province of Gascony increased. (E) The population of medieval France increased substantially during the first five decades of the 1300s, until the deadly bubonic plague decimated the population of France after 1348. | 89. Rhizobium bacteria living in the roots of bean plants or other legumes produce fixed nitrogen, which is one of the essential plant nutrients and which for non-legume crops, such as wheat, normally must be supplied by applications of nitrogen-based fertilizer. So if biotechnology succeeds in producing wheat strains whose roots will play host to Rhizobium bacteria, the need for artificial fertilizers will be reduced. The argument above makes which one of the following assumptions? (A) Biotechnology should be directed toward producing plants that do not require artificial fertilize. (B) Fixed nitrogen is currently the only soil nutrient that must be supplied by artificial fertilizer for growing wheat crops. (C) There are no naturally occurring strains of wheat or other grasses that have Rhizobium bacteria living in their roots. (D) Legumes are currently the only corps that produce their own supply of fixed nitrogen. (E) Rhizobium bacteria living in the roots of wheat would produce fixed nitrogen. 90. Current legislation that requires designated sections for smokers and nonsmokers on the premises of privately owned businesses is an intrusion into the private sector that cannot be justified. The fact that studies indicate that nonsmokers might be harmed by inhaling the smoke from others’ cigarettes is not the main issue. Rather, the main issue concerns the government’s violation of the right of private businesses to determine their own policies and rules. Which of the following is a principle that, if accepted, could enable the conclusion to be properly drawn? (A) Government intrusion into the policies and rules of private businesses is justified only when individuals might be harmed. (B) The right of individuals to breathe safe air supersedes the right of businesses to be free from government intrusion. (C) The right of businesses to self-determination overrides whatever right or duty the government may have to protect the individual. (D) It is the duty of private businesses to protect employees from harm in the workplace. (E) Where the rights of businesses and the duty of government conflict, the main issue is finding a successful compromise. Test 6 91. Leachate is a solution, frequently highly contaminated, that develops when water permeates a landifill’s site. If and only if the landfill’s capacity to hold liquids is exceeded does the leachate escape into the environment, generally in unpredictable queantities. A method must be found for disposing of leachate. Most landfill leachate is sent directly to sewage treatment plants, but not all sewage plants are capable of handling the highly contaminated water. Which one of the following can be inferred from the passage? (A) The ability to predict the volume of escaping landfill leachate would help solve the disposal problem. (B) If any water permeates a landfill, leachate will escape into the environment. (C) No sewage treatment plants are capable of handling leachate. (D) Some landfill leachate is sent to sewage treatment plants that are incapable of handling it. (E) If leachate does not escape from a landfill into the environment, then the landfill’s capacity to hold liquids has not been exceeded. 92. Baking for winter holidays is a tradition that may have a sound medical basis. In midwinter, when days are short, many people suffer from a specific type of seasonal depression caused by lack of sunlight. Carbohydrates, both sugars and starches, boost the brain’s levels of serotonin, a neurotransmitter that improves the mood. In this respect, carbohydrates act on the brain in the same way as some antidepressants. Thus, eating holiday cookies may provide an effective form of self-prescribed medication. Which one of the following can be properly inferred from the passage? (A) Seasonal depression is one of the most easily treated forms of depression. (B) Lack of sunlight lowers the level of serotonin in the brain. (C) People are more likely to be depressed in midwinter than at other times of the year. (D) Some antidepressants act by changing the brain’s level of serotonin. (E) Raising the level of neurotransmitters in the brain effectively relieves depression. 93. The efficiency of microwave ovens in destroying the harmful bacteria frequently found in common foods is diminished by the presence of salt in the food being cooked. When heated in a microwave oven, the interior of unsalted food reaches temperatures high enough to kill bacteria that cause food poisoning, but the interior of salted food does not. Scientists theorize that salt effectively blocks the microwaves from heating the interior. Which one of the following conclusions is most supported by the information above? (A) The kinds of bacteria that cause food poisoning are more likely to be found on the exterior of food than in the interior of food. (B) The incidence of serious food poisoning would be significantly reduced if microwave ovens were not used by consumers to cook or reheat food. (C) The addition of salt to food that has been cooked or reheated in a microwave oven can increase the danger of food poisoning. (D) The danger of food poisoning can be lessened if salt is not used to prepare foods that are to be cooked in a microwave oven. (E) Salt is the primary cause of food poisoning resulting from food that is heated in microwave ovens. 94. When 100 people who have not used cocaine are tested for cocaine use, on average only 5 will test positive. By contrast, of every 100 people who have used cocaine 99 will test positive. Thus, when a randomly chosen group of people is tested for cocaine use, the vast people is tested for cocaine use, the vast majority of those who test positive will be people who have used cocaine. A reasoning error in the argument is that the argument (A) attempts to infer a value judgement from purely factual premises (B) attributes to every member of the population the properties of the average member of the population (C) fails to take into account what proportion of the population have used cocaine (D) ignores the fact that some cocaine users do not test positive (E) advocates testing people for cocaine use when there is no reason to suspect that they have used cocaine 95. Scientific research that involves international collaboration has produced papers of greater influence, as measured by the number of times a paper is cited in subsequent papers, than has research without any collaboration. Papers that result from international collaboration are cited an average of seven times, whereas papers with single authors are cited only three times on average. This difference shows that research projects conducted by international research teams conducted by single researchers. Which one of the following is an assumption on which the argument depends? (A) Prolific writers can inflate the number of citations they receive by citing themselves in subsequent papers. (B) It is possible to ascertain whether or not a paper is the product of international collaboration by determining the number of citations it has received. (C) The number of citations a paper receives is a measure of the importance of the research it reports. (D) The collaborative efforts of scientists who are citizens of the same country do not produce papers that are as important as papers that are produced by international collaboration. (E) International research teams tend to be more generously funded than are single researchers. 96. Prominent business executives often play active roles in United States presidential campaigns as fundraisers or backroom strategists, but few actually seek to become president themselves. Throughout history the great majority of those who have sought to become president have been lawyers, military leaders, or full-time politicians. This is understandable, for the personality and skills that make for success in business do not make for success in politics. Business is largely hierarchical, whereas politics is coordinative. As a result, business executives tend to be uncomfortable with compromises and power-sharing, which are inherent in politics. Which one of the following, if true, most seriously weaken the proposed explanation of why business executives do not run for president? (A) Many of the most active presidential fundraisers and backroom strategists are themselves politicians. (B) Military leaders are generally no more comfortable with compromises and power-sharing than are business executives. (C) Some of the skills needed to become a successful lawyer are different from some of those needed to become a successful military leader. (D) Some former presidents have engaged in business ventures after leaving office. (E) Some hierarchically structured companies have been major financial supporters of candidates for president. 97. A scientific theory is a good theory if it satisfies two requirements: it must accurately describe a large class of observations in terms of a model that is simple enough to contain only a few elements, and it must make definite predictions about the results of future observations. For example, Aristotle’s cosmological theory, which claimed that everything was made out of four elements---earth, air, fire, and water---satisfied the first requirement, but it did not make any definite predictions. Thus, Aristotle’s cosmological theory was not a good theory. If all the statements in the passage are true, each of the following must also be true EXCEPT: (A) Prediction about the results of future observations must be made by any good scientific theory (B) Observation of physical phenomena was not a major concern in Aristotle’s cosmological theory. (C) For elements can be the basis of a scientific model that is simple enough to meet the simplicity criterion of a good theory. (D) A scientific model that contains many elements is not a good theory. (E) Aristotle’s cosmological theory described a large class of observations in terms of only four elements. 98. Compared to nonprofit hospitals of the same size, investor-owned hospitals require less public investment in the form of tax breaks, use fewer employees, and have higher occupancy levels. It can therefore be concluded that investor-owned hospitals are a better way of delivering medical care than are nonprofit hospitals. Which one of the following, if true, most undermines the conclusion drawn above? (A) Nonprofit hospitals charge more per bed than do investor-owned hospitals. (B) Patients in nonprofit hospitals recover more quickly than do patients with comparable illnesses in investor-owned hospitals. (C) Nonprofit hospitals do more fundraising than do investor-owned hospitals. (D) Doctors at nonprofit hospitals earn higher salaries than do similarly-qualified doctors at investor-owned hospitals. (E) Nonprofit hospitals receive more donations than do investor-owned hospital. 99. Concetta: Franchot was a great writer because she was ahead of her time in understanding that industrialization was taking an unconscionable toll on the family structure of the working class. Alicia: Franchot was not a great writer. The mark of a great writer is the ability to move people with the power of the written word, not the ability to be among the first to grasp a social issue. Besides, the social consequences of industrialization were widely understood in Franchot’s day. In her disagreement with Concetta, Alicia does which one of the following? (A) accepts Concetta’s criterion and then adds evidence to Concetta’s case (B) discredits Concetta’s evidence and then generalizes from new evidence (C) rejects Concetta’s criterion and then disputes a specific claim (D) disputes Concetta’s conclusion and then presents facts in support of an alternative creterion (E) attacks one of Concetta’s claims and then criticizes the structure of her argument Questions 100-101 Zelda: Dr. Ladlow, a research psychologist, has convincingly demonstrated that his theory about the determinants of rat behavior generates consistently accurate predictions about how rats will perform in a maze. On the basis of this evidence, Dr. Ladlow has claimed that his theory is irrefutably correct. Anson: Then Dr. Ladlow is not a responsible psychologist, Dr. Ladlow’s evidence does not conclusively prove that his theory is correct. Responsible psychologists always accept the possibility that new evidence will show that their theories are incorrect. 100. Which one of the following can be properly inferred from Anson’s argument? (A) Dr. Ladlow’s evidence that his theory generates consistently accurate predictions about how rats will perform in a maze is inaccurate. (B) Psychologists who can derive consistently accurate predictions about how rats will perform in a maze from their theories cannot responsibly conclude that those theories cannot be disproved. (C) No matter how responsible psychologists are, they can never develop correct theoretical explanations. (D) Responsible psychologists do not make predictions about how rats will perform in a maze. (E) Psychologists who accept the possibility that new evidence will show that their theories are incorrect are responsible psychologists. 101. Anson bases his conclusion about Dr. Ladlow on which one of the following? (A) an attack on Dr. Ladlow’s character (B) the application of a general principle (C) the use of an ambiguous term (D) the discrediting of facts (E) the rejectiion of a theoretical explanation Questions 102-103 The brains of identical twins are genetically identical. When only one of a pair of identical twins is a schizophrenic, certain areas of the affected twin’s brain are smaller than corresponding areas in the brain of the unaffected twin. No such differences are found when neither twin is schizophrenic. Therefore, this discovery provides definitive evidence that schizophrenia is caused by damage to the physical structure of the brain. 102. Which one of the following is an assumption required by the argument? (A) The brain of a person suffering from schizophrenia is smaller than the brain of anyone not suffering from schizophrenia. (B) The relative smallness of certain parts of the brains of schizophrenics is not the result of schizophrenia or of medications used in its treatment. (C) The brain of a person with an identical twin is no smaller, on average, than the brain of a person who is not a twin. (D) When a pair of identical twins both suffer from schizophrenia, their brains are the same size. (E) People who have an identical twin are no more likely to suffer from schizophrenia than those who do not. 103. If the statements on which the conclusion above is based are all true, each of the following could be true EXCEPT: (A) People who lack a genetic susceptibility for the disease will not develop schizophrenia (B) Medications can control most of the symptoms of schizophrenics share many of the characteristics found in those of people without the disorder (C) The brains of schizophrenics share many of the characteristics found in those of people without the disorder (D) It will eventually be possible to determine whether or not someone will develop schizophrenia on the basis of genetic information alone (E) Brain abnormalities associated with schizophrenia are the result of childhood viral infections that inhibit the development of brain cells 104. That the policy of nuclear deterrence has worked thus far is unquestionable. Since the end of the Second World War, the very fact that there were nuclear armaments in existence has kept major powers from using nuclear weapons, for fear of starting a worldwide nuclear exchange that would make the land of the power initiating it uninhabitable. The proof is that a third world war between superpowers has not happened. Which one of the following, if true, indicates a flaw in the argument? (A) Maintaining a high level of nuclear armaments represents a significant drain on a country’s economy. (B) From what has happened in the past, it is impossible to infer with certainty what will happen in the future, so an accident could still trigger a third world war between superpowers. (C) Continuing to produce nuclear weapons beyond the minimum needed for deterrence increase the likelihood of a nuclear accident. (D) The major powers have engaged in many smaller-scale military operations since the end of the Second World War, while refraining from a nuclear confrontation. (E) It cannot be known whether it was nuclear deterrence that worked, or some other factor, such as a recognition of the economic value of remaining at peace. 105. A survey of alumni of the class of 1960 at Aurora University yielded puzzling results. When asked to indicate their academic rank, half of the respondents reported that they were in the top quarter of the graduating class in 1960. Which one of the following most helps account for apparent contradiction above? (A) A disproportionately large number of high ranking alumni responded to the survey. (B) Few, if any, respondents were mistaken about their class rank. (C) Not all the alumni who were actually in the top quarter responded to the survey. (D) Almost all of the alumni who graduated in 1960 responded to the survey. (E) Academic rank at Aurora University was based on a number of considerations in addition to average grades. 106. The seventeenth-century physicist Sir Isaac Newton is remembered chiefly for his treatises on motion and gravity. But Newton also conducted experiments secretly for many years abased on the arcane theories of alchemy, trying unsuccessfully to transmute common metals into gold and produce rejuvenating elixirs. If the alchemists of the seventeenth century had published the results of their experiments, chemistry in the eighteenth century would have been more advance than it actually was. Which one of the following assumptions would allow the conclusion concerning eighteenth-century chemistry to be properly drawn? (A) Scientific progress is retarded by the reluctance of historians to acknowledge the failures of some of the great scientists. (B) Advances in science are hastened when reports of experiments, whether successful or not, are available for review by other scientists. (C) Newton’s work on motion and gravity would not have gained wide acceptance if the results of his work in alchemy had also been made public. (D) Increasing specialization within the sciences makes it difficult for scientists in one field to understand the principles of other fields. (E) The seventeenth-century alchemists could have achieved their goal only if their experiments had been subjected to public scrutiny. 107. Because of the recent transformation of the market, Quore, Inc., must increase the next two years, or it will certainly go bankrupt. In fact, however, Quore’s production structure is such that if a 10 percent productivity increase is possible, then a 20 percent increase is attainable. If the statements above are true, which one of the following must on the basis of them also be true? (A) It is only Quore’s production structure that makes it possible for Quore to survive the transformation of the market. (B) Quore will not go bankrupt if it achieves a productivity increase of 20 percent over the next two years. (C) If the market had not been transformed, Quore would have required no productivity increase in order to avoid bankruptcy. (D) Because of the transformation of the market, Quore will achieve a productivity increase of 10 percent over the next two years. (E) If a 20 percent productivity increase is unattainable for Quore, then it must go bankrupt. 108. Terry: If you want to get a decent job, you should go to college. Mark: That is not true. There are other reasons to go to college than wanting to get a good job. Mark’s response shows that he interpreted Terry’s remarks to mean that (A) college is one of many places to get trained for a job (B) decent jobs are obtained only by persons who have gone to college (C) wanting to get a decent job is the only reason for going to college (D) training for decent jobs is available only at colleges (E) all people who want decent jobs go to college                                                                           Test 7 109. Several studies have shown that hospitals are not all equally successful: patients are much more likely to die in some of them than in others. Since the hospitals in the studies had approximately equal per-patient funding, differences in the quality of care provided by hospital staff are probably responsible for the differences in mortality rates. Which one of the following, if true, casts the most doubt on the conclusion drawn above? (A) The staff in some of the hospital studies had earned more advanced degrees, on average, than the staff in the other hospitals. (B) Patient populations vary substantially in average severity of illness from hospital to hospital. (C) The average number of years that staff members stay on at a given job varies considerably from one hospital to another. (D) Approximately the same surgical procedures were performed in each of the hospitals covered in the studies. (E) Mortality rates for hospital patients do not vary considerably from one region of the country to another. 110. Animals with a certain behavioral disorder have unusually high levels of aluminum in their brain tissue. Since a silicon-based compound binds to aluminum and prevents it from affecting the brain tissue, animals can be cured to the disorder by being treated with the compound. The argument is based on which one of the following assumptions? (A) Animals with the disorder have unusually high but invariable levels of aluminum in their brain tissue. (B) Aluminum is the cause of the disorder rather than merely an effect of it. (C) Introducing the compound into the brain tissue has no side effects. (D) The amount of the compound needed to neutralize the aluminum in an animal’s brain tissue varies depending upon the species. (E) Aluminum is never present in normal brain tissue. 111. The stated goal of the government’s funding program for the arts is to encourage the creation of works of artistic excellence. Senator Beton claims, however, that a government-funded artwork can never reflect the independent artistic conscience of the artist because artists, like anyone else who accepts financial support, will inevitably try to please those who control the distribution of that support. Senator Beton concludes that government funding of the arts not only is a burden on taxpayers but also cannot lead to the creation of works of true artistic excellence. Which one of the following is an assumption on which Senator Beton’s argument is based? (A) Most taxpayers have little or no interest in the creation of works of true artistic excellence. (B) Government funding of the arts is more generous than other financial support most artists receive. (C) Distribution of government funds for the arts is based on a broad agreement as to what constitutes artistic excellence. (D) Once an artist has produced works of true artistic excellence, he or she will never accept government funding. (E) A contemporary work of art that does not reflect the independent artistic conscience of the artist cannot be a work of true artistic excellence. 112. Older United States automobiles have been identified as contributing disproportionately to global air pollution. The requirement in many jurisdictions that automobiles pass emission-control inspections has had the effect of taking many such automobiles out of service in the United States, as they fail inspection and their owners opt to buy newer automobiles. Thus the burden of pollution such older United States automobiles contribute to the global atmosphere will be gradually reduced over the next decade. Which one of the following, if true, most seriously weakens the argument? (A) It is impossible to separate the air of one country or jurisdiction from that of others. Since air currents circle the globe. (B) When automobiles that are now new become older, they will, because of a design change, cause less air pollution than older automobiles do now. (C) There is a thriving market for used older Untied States automobiles that are exported to regions that have no emission-control regulations. (D) The number of jurisdictions in the United States requiring automobiles to pass emission-control inspections is no longer increasing. (E) Even if all the older automobiles in the United States were retired from service, air pollution from United States automobiles could still increase if the total number of automobiles in use should increase significantly. 113. The reforms to improve the quality of public education that have been initiated on the part of suppliers of public education have been insufficient. Therefore, reforms must be demanded by consumers. Parents should be given government vouchers with which to pay for their children’s education and should be allowed to choose the schools at which the vouchers will be spent. To attract students, academically underachieving schools will be forced to improve their academic offerings. The argument assumes that (A) in selecting schools parents would tend to prefer a reasonable level of academic quality to greater sports opportunities or more convenient location. (B) improvement in the academic offerings of schools will be enforced by the discipline of the job market in which graduation students compete (C) there is a single best way to educate students (D) children are able to recognize which schools are better and would influence their parents’ decisions. (E) schools would each improve all of their academic offering and would not tend to specialize in one particular field to the exclusion of others. 114. The number of North American children who are obese-that is, who have more body fat than do 85 percent of North American children their age---is steadily increasing, according to four major studies conducted over the past 15 years. If the finding reported above is correct, it can be properly concluded that (A) when four majors studies all produce similar results, those studies must be accurate (B) North American children have been progressively less physically active over the past 15 years (C) the number of North American children who are not obese increased over the past 15 years (D) over the past 15 years, the number of North American children who are underweight has declined (E) the incidence of obesity in North American children tends to increase as the children grow older 115. School superintendent: It is a sad fact that, until now, entry into the academically best high school in our district has been restricted to the children of people who were wealthy enough to pay the high tuition. Parents who were previously denied the option of sending their children to this school now have the option, since I am replacing the tuition requirement with a requirement that allows only those who live in the neighborhood of the school to attend. The superintendent’s claim about the effect of replacing the tuition requirement relies on the assumption that (A) the residents of the school’s neighborhood tent to be wealthy (B) people other than those wealthy enough to have paid the old tuition are able to live in the neighborhood of the school (C) people less wealthy than those who were able to pay the old tuition are in the majority in the district (D) there are no high schools in the district other than the one referred to by the superintendent (E) there are many people not wealthy enough to have paid the old tuition who wish to have their children attend the school 116. Approximately 7.6 million women who earn incomes have preschool-age children, and approximately 6.4 million women are the sole income earners for their families. These figures indicate that there are comparatively few income-earning women who have preschool-age children but are not the sole income earners for their families. A major flaw in the reasoning is that it (A) relies on figures that are too imprecise to support the conclusion drawn (B) overlooks the possibility that there is little or no overlap between the two populations of women cited (C) fails to indicate whether the difference between the two figures cited will tend to remain stable over time (D) ignores the possibility that families with preschool-age children might also have older children (E) provides no information on families in which men are the sole income earners 117. A certain viral infection is widespread among children, and about 30 percent of children infected with the virus develop middle ear infections. Antibiotics, although effective in treating bacterial infections, have no effect on the virus. Yet when middle ear infections in children infected with the virus are treated with antibiotics, the ear often clear up. Which one of the following most helps to explain the success of the treatments with antibiotics? (A) Although some types of antibiotics fail to clear up certain infections, other types of antibiotics might provide effective treatment for those infections. (B) Children infected with the virus are particularly susceptible to bacterial that infect the middle age. (C) Many children who develop middle ear infections are not infected with the virus. (D) Most viral infections are more difficult to treat than are most bacterial infections. (E) Among children not infected with the virus, fewer than 30 percent develop middle ear infections. 118. There are about 75 brand of microwave popcorn on the market altogether, they account for a little over half of the money from sales of microwave food products. It takes three minutes to pop corn in the microwave, compared to even minutes to pop corn conventionally. Yet by weight, microwave popcorn typically costs over five times as much as conventional popcorn. Judging by the popularity of microwave popcorn, many people are willing to pay a high price for just a little additional convenience. Which one of the following statements, if true, would call into question the conclusion in the passage? (A) More than 50 percent of popcorn purchasers buy conventional popcorn rather than microwave popcorn. (B) Most people who prefer microwave popcorn do so because it is less fattening than popcorn that is popped conventionally in oil. (C) The price of microwave popcorn reflects its packaging more than it reflects the quality of the popcorn contained in the package. (D) The ratio of unpopped kernels to popped kernels is generally the same whether popcorn is popped in a microwave or conventionally in oil. (E) Because microwave popcorn contains additives not contained in conventional popcorn, microwave popcorn weights more than an equal volume of conventional popcorn. 119. For a television program about astrology, investigators went into the street and found twenty volunteers born under the sign of Gemini who were willing to be interviewed on the program and to take a personality test. The test confirmed the investigators’ personal impressions that each of the volunteers was more sociable and extroverted than people are on average. This modest investigation thus supports the claim that one’s astrological birth sign influences one’s personality. Which one of the following, if true, indicates the most serious flaw in the method used by the investigators? (A) The personality test was not administered or scored personally by the investigators. (B) People born under astrological signs other than Gemini have been judged by astrologers to be much less sociable than those born under Gemini. (C) The personal impressions the investigators first formed of other people have tended to be confirmed by the investigators’ late experience of those people. (D) There is not likely to be a greater proportion of people born under the sign of Gemini on the street than in the population as a whole. (E) People who are not sociable and extroverted are not likely to agree to participate in such an investigation. 120. In Europe, schoolchildren devote time during each school day to calisthenics. North American schools rarely offer a daily calisthenics program. Tests prove that North America children are weaker, slower, and shorter-winded than European children. We must conclude that North American children can be made physically fit only if they participate in school calisthenics on a daily basis. Which one of the following is assumed in the passage? (A) All children can be made physically fit by daily calisthenics. (B) All children can be made equally physically fit by daily calisthenics. (C) Superior physical fitness produces superior health. (D) School calisthenics are an in indispensable factor in European children’s (E) North American children can learn to eat a more nutritious diet as well as to exercise daily. 121. Observatory director: Some say that funding the megatelescope will benefit only the astronomers who will work with it. This dangerous point of view, applied to the work of Maxwell, Newton, or Einstein, would have stified their research and deprived the world of beneficial applications, such as the development of radio, that followed from that research. If the statements above are put forward as an argument in favor of development of the megatelescope, which one of the following is the strongest criticism of that argument? (A) It appeals to the authority of experts who cannot have known all the issues involved in construction of the megatelescope. (B) It does not identify those opposed to development of the megatelescope. (C) It launches a personal attack on opponents of the megatelescope by accusing them of having a dangerous point of view. (D) It does not distinguish between the economic and the intellectual senses of “benefit”. (E) It does not show that the proposed megatelescope research is worthy of comparison with that of eminent scientists in its potential for applications. 122. Photovoltaic power plants produce electricity from sunlight. As a result of astonishing recent technological advances, the cost of producing electric power at photovoltaic power plants, allowing for both construction and operating costs, it one-tenth of what it was 20 years ago, whereas the corresponding cost for traditional plants, which burn fossil fuels, has increased. Thus, photovoltaic power plants offer a less expensive approach to meeting demand for electricity than do traditional power plants. The conclusion of the argument is properly drawn it which one of the following is assumed? (A) the cost of producing electric power at traditional plants has increased over the past 20 years. (B) Twenty years ago, traditional power plants were producing 10 times more electric power than were photovoltaic plants. (C) None of the recent technological advances in producing electric power at photovoltaic plants can be applied to producing power at traditional plants. (D) Twenty years ago, the cost of producing electric power at photovoltaic plants was less than 10 times the cost of producing power at traditional plants. (E) The cost of producing electric power at photovoltaic plants is expected to decrease further, while the cost of producing power at traditional plants is not expected to decrease. 123. When a group of children who have been watching television programs that include acts of violence is sent to play with a group of children who have been watching programs that do not include acts of violence, the children who have been watching violent programs commit a much greater number of violent acts in their play than do the children who have been watching nonviolent programs. Therefore, children at play can be prevented from commiting violent acts by not being allowed to watch violence on television. The argument in the passage assumes which one of the following? (A) Television has a harmful effect on society. (B) Parents are responsible for the acts of their children. (C) Violent actions and passive observation of violent actions are not related. (D) There are no other differences between the two groups of children that might account for the difference in violent behavior. (E) Children who are treated violently will respond with violence. 124. It is repeatedly claimed that the dumping of nuclear waste poses no treat to people living nearby. If this claim could be made with certainty, there would be no reason for not locating sites in areas of dense population. But the policy of dumping nuclear waste only in the more sparsely populated regions indicates, at the very least, some misgiving about safety on the part of those responsible for policy. Which one of the following, if true, would most seriously weaken the argument? (A) Evacuation plants in the event of an accident could not be guaranteed to work perfectly except where the population is small. (B) In the event of an accident, it is certain that fewer people would be harmed in a sparsely populated than in a densely populated area. (C) Dumping of nuclear waste poses fewer economic and bureaucratic problems in sparsely populated than in densely populated areas. (D) There are dangers associated with chemical waste, and it, too, is dumped away from areas of dense population. (E) Until there is no shred of doubt that nuclear dumps are safe, it makes sense to situate them where they pose the least threat to the public. 125. A society’s infant mortality rate is an accepted indicator of that society’s general health status. Even though in some localities in the United States the rate is higher than in many developing countries. In the United States, overall the rate has been steadily declining. This declines does not necessarily indicate, however, that babies in the United States are now, on the average, healthier at birth than they were in the past. Which one of the following reasons, if true, most strongly supports the claim made above about the implications of the decline? (A) The figure for infant mortality is compiled as an overall rate and thus masks deficiencies in particular localities. (B) Low birth weight is a contributing factor in more than half of the infant deaths in the United States. (C) The United States has been developing and his achieved extremely sophisticated technology for saving premature and low-birth-weight babies, most of whom require extended hospital stays. (D) In eleven states of the United States, the infant mortality rate declined last year. (E) Babies who do not receive adequate attention from a caregiver fail to thrive and so they gain weight slowly. 126. The public is well aware that high blood cholesterol levels raise the risk of stroke caused by blood clots, but a recent report concludes that people with low blood cholesterol levels are at increased risk of the other lethal type of stroke---cerebral hemorrhage, caused when a brain artery busts. The report suggests that because blood cholesterol plays a vita role in maintaining cell membranes, low blood cholesterol weakens artery walls, making them prone to rupture. The conclusion thus supports a long-standing contention by Japanese researchers that Western diets better protect against cerebral hemorrhage than do non-Western diets. The argument is based on which one of the following assumption? (A) Western diets are healthier than non-Western diets. (B) Western diets result in higher blood cholesterol levels than do non-Western diets. (C) High blood cholesterol levels preclude the weakening of artery walls. (D) Cerebral hemorrhages are more dangerous than strokes caused by blood clots. (E) People who have low blood pressure are at increased risk of cerebral hemorrhage. Test 8 127. Anthropologists assert that cultures advance only when independence replaces dependence---that is, only when imposition by outsiders is replaced by initiative from within. In other words, the natives of a culture are the only ones who can move that culture foward. Non-natives may provide valuable advice, but any imposition of their views threatens independence and thus progress. If one looks at individual schools as separate cultures, therefore, the key to educational progress is obvious: which one of the following best completes the passage? (A) individual schools must be independent of outside imposition (B) some schools require more independence than others, depending on the initiative of their staffs and students (C) school systems officials must tailor their initiatives for change to each individual school in the system (D) outsiders must be prevented from participation in schools’ efforts to advance (E) the more independent a school is, the more educational progress it will make 128. The public in the United States has in the past been conditioned to support a substantial defense budget by the threat of confrontation with the Eastern bloc. Now that that threat is dissolving, along with the Eastern bloc itself, it is doubtful whether the public can be persuaded to support an adequate defense budget. Which one of the following indicates a weakness in the position expressed above? (A) It presupposes that public opinion can be manipulated indefinitely, without the public’s becoming aware of that manipulation. (B) It refers to past and present events that do not have a causal connection with public support of the budget. (C) It assumes as fact what it seeks to establish by (D) It fails to give any reason for the judgement it reaches. (E) It hinges on the term “adequate”, the precise meaning of which requires reevaluation in the new context. 129. Behind the hope that computers can replace teachers is the idea that the student’s understanding of the subject being taught consists in knowing facts and rules, the job of a teacher being to make the facts and rules explicit and convey them to the students, either by practice drills or by coaching. If that were indeed the way the mind works, the teacher could transfer facts and rules to the computer, which could replace the teacher as drillmaster and coach. But since understanding does not consist merely of knowing facts and rules, but of the grasp of the general concepts underlying them, the hope that the computer will eventually replace the teacher is fundamentally misguided. Which one of the following, if true, would most seriously undermine the author’s conclusion that computers will not eventually be able to replace teachers? (A) Computers are as good as teachers at drilling students on facts and rules. (B) The job of a teacher is to make students understand the general concepts underlying specific facts and rules. (C) It is possible to program computers so that they can teach the understanding of general concepts that underlie specific facts and rules. (D) Because they are not subject to human error, computers are better than teachers at conveying facts and rules. (E) It is not possible for students to develop an understanding of the concepts underlying facts and rules through practice drills and coaching. 130. The mind and the immune system have been shown to be intimately linked, and scientists are consistently finding that doing good deeds benefits one’s immune system. The bone marrow and spleen, which produce the white blood cells needed to fight infection, are both connected by neural pathways to the brain. Recent research as shown that they activity of these white blood cells is stimulated by beneficial chemicals produced by the brain as a result of magnanimous behavior. The statements above, if true, support the view that (A) good deeds must be based on unselfish motives lack of magnanimity is the cause of most serious illnesses. (B) magnanimous behavior can be regulated by the presence of absence of certain chemicals in the brain. (C) magnanimous behavior can be regulated by the presence or absence of certain chemicals in the brain. (D) magnanimity is beneficial to one’s own interests (E) the number of white blood cells will increase radically if behavior is consistently magnanimous 131. To become an expert on a musical instrument, a person must practice. If a person practice a musical instrument for three hours each day, they will eventually become expert on that instrument. Therefore, if a person is an expert on a musical instrument, that person must have practiced for at least three hours each day. Which one of the following most accurately describes a flaw in the reasoning above? (A) The conclusion fails to take into account that people who practice for three hours every day might not yet have reached a degree of proficiency that everyone would consider expert. (B) The conclusion fails to take into account that practicing for less than three hours each day may be enough for some people to become experts. (C) the conclusion fails to take into account that if a person has not practiced for at least three hours a day, the person has not become an expert. (D) The conclusion fails to take into account that three consecutive hours of daily practice is not recommended by all music teachers. (E) The conclusion fails to take into account that few people have the spare time necessary to devote three hours daily to practice. Questions 132-133 Nature constantly adjusts the atmospheric carbon level. An increase in the level causes the atmosphere to hold more heat, which causes more water to evaporate from the oceans, which causes increased rain. Rain washes some carbon from the air into the oceans, where it eventually becomes part of the seabed. A decrease in atmosphere carbon causes decreased evaporation from the oceans, which cause less rain, and thus less carbon is washed into the oceans. Yet some environmentalists worry that burning fossil fuels may raise atmospheric carbon to a dangerous level. It is true that a sustained increase would threaten human life. But the environmentalists should relax---nature will continually adjust the carbon level. 132. Each of the following can be inferred from the information in the passage EXCEPT: (A) A decrease in the level of atmospheric heat causes a decrease in the amount of carbon that rain washes into the oceans from the air. (B) An increase in the level of carbon in the atmosphere causes increased evaporation of ocean water. (C) An increase in the level of atmospheric heat causes increased rainfall. (D) A decrease in the level of carbon in the atmosphere causes decreased evaporation of ocean water. (E) A decrease in the level of atmospheric heat causes a decrease in the level of carbon in the atmosphere. 133. Which one of the following, if true, would most weaken the argument in the passage? (A) Plant life cannot survive without atmospheric carbon. (B) It is not clear that breathing excess carbon in the atmosphere will have a negative effect on human life. (C) Carbon is part of the chemical “blanket” that keeps the Earth warm enough to sustain human life. (D) Breathing by animals releases almost 30 times as much carbon as does the burning of fossil fuels. (E) The natural adjustment process, which occurs over millions of years, allow wide fluctuations in the carbon level in the short term. 134. The more television children watch, the less competent they are in mathematical knowledge. More than a third of children in the United States watch television for more than five hours a day; in South Korea the figure is only 7 percent. But whereas less than 15 percent of children in the United States understand advanced measurement and geometric concepts, 40 percent of South Korean children are competent in these areas. Therefore, if United States children are to do well in mathematics, they must watch less television. Which one of the following is an assumption upon in advanced measurement and geometric concepts than are South Korean children? (A) Children in the United States are less interested in advanced measurement and geometric concepts than are South Korean children. (B) South Korean children are more disciplined about doing schoolwork than are (C) Children who want to do well in advanced measurement and geometry will watch less television. (D) A child’s ability in advanced measurement and geometry increases if he or she watches less than one hour of television a day. (E) The instruction in advanced measurement and geometric concepts available to children in the United States is not substantially worse than that available to South Korean children. 135. In a new police program, automobile owners in some neighborhoods whose cars are not normally driven between 1 A.M. and 5 A.M. can display a special decal in the cars’ windows and authorize police to stop the cars during those hours to check the drivers’ licenses. The theft rate for cars bearing such decals is much lower than had been usual for cars in those neighborhoods. If it is concluded from the statements above that automobile theft has been reduced by the program, which one of the following would be most important to answer in evaluating that conclusion? (A) Are owners who are cautious enough to join the program taking other special measures to protect their cars against theft? (B) In how many neighborhoods is the police program operating? (C) Are cars in neighborhoods that are actively participating in the program sometimes stolen during daylight hours? (D) Will owners who have placed decals on their cars’ windows but who find it necessary to drive between 1 A.M. and 5 A. M. Be harassed by police? (E) Are the neighborhoods in which the program has been put into effect a representative cross section of neighborhoods with respect to the types of automobiles owned by residents? : 1-5 DCCBA 6-10 ECBAB 11-15 BECBC 16-20 BCEBC 21-25 BBAEC 26-30 CEABD 31-35 BBCDD 36-40 BAAEA 41-45 ECBCB 46-50 EACBC 51-55 BCCDC 56-60 EAEAE 61-65 CBDDC 66-70 ACBAB 71-75 ECABC 76-80 BDAED 81-85 CADDD 86-90 ADAEC 91-95 EDDCC 96-100 BBBCB 101-105 BBDEA 106-110 BECBB 111-115 ECACB 116-120 BBBED 121-125 EDDCC 126-130 BAECD 131-135 BEEEA PAGE 51
/
本文档为【Feifei_Logic】,请使用软件OFFICE或WPS软件打开。作品中的文字与图均可以修改和编辑, 图片更改请在作品中右键图片并更换,文字修改请直接点击文字进行修改,也可以新增和删除文档中的内容。
[版权声明] 本站所有资料为用户分享产生,若发现您的权利被侵害,请联系客服邮件isharekefu@iask.cn,我们尽快处理。 本作品所展示的图片、画像、字体、音乐的版权可能需版权方额外授权,请谨慎使用。 网站提供的党政主题相关内容(国旗、国徽、党徽..)目的在于配合国家政策宣传,仅限个人学习分享使用,禁止用于任何广告和商用目的。

历史搜索

    清空历史搜索